You are on page 1of 63
CHAPTER 4 Direct-Current Generators 4.0 Introdu Wet geese tating machin wit the direct-current generator. Direct-current generators are not as common as they used (0 be, hecause direct current, when required, is mainly produced by electronic rectifiers ‘can convert the current of an ae system into direct without any moving Nevertheless. an understanding of d important because it represents a logical introduc tion the behavior of de motors, Indeed. many de ‘motors in industry actually operate as generators for briet perio Commercial de generators and motors are built the same way: consequently, any de ‘operate as a motor and vige versa, Owing to their similar construction, the fundamental properties of generators and motors are identi anything we learn about a de genera rectly applied to a de moror Inthis chapter we begin with the basic principles of a 2-pole generator when it operates at no-lou We show the importance of brush position and de- fine what is meant by the newstul point. We show how the induced voltage is generated and what de current using pars erator ean n This is followed by a study of the behavior of the generator under loud, Mechanical torque, direction of ‘current flow, and the importance of armature reaction are discussed. The need for commutating poles and the problem of pole-tip saturation are covered next. ‘We then discuss the major types of de generators and their voltage-r The chapter ends with a description of the actual physical construction of direct-current machines, ng multipole designs. 4.1 Generating an ac voltage Irrelevant as it may seem, the study of a divect- current (de) jor has (© begin with a know! edge of the ulternating-current (ac) generator. The reason is that the voltage generated in any de gen: erator is inherently alternating and only bevomes de after it has been rectified by the commutator Fig. 4.1 shows an elementary ac generator composed of a coil that revolves at 60 r/min be ween the N, § poles of a permanent magnet. The rotation is due to an external driving force, such a8 a motor (not shown), The coil is connected 10 ‘Wo slip rings mounted on the shat. The slip rings are connected to an external load by means of 1Wo stationary brushes x and y. 72 ELECTRICAL MACHINES AND TRANSFORMERS, covme Figure 4.1 ‘Schematic diagram of an elementary ac generator turning at 1 revolution per second. As the evil rotates. voltage is induced (Ey. borwevn is terminals 4 and D. This voltage appears between the brushes and, therefore, acrows the load The voltage is generated beciuse the conductors of the coil eut across the flux produced by the N.S poles, The induced vollage is therefore maximum (20 V, say) when the coil is momentarily inthe hori- otal position, as shown. No Has és Cut When the ccoif is momentarily in the vertical position; conse ly the voltage at these instants is zero. Another e is that its polarity changes every time the coil makes half a turn, The voltage ean therefore be represented asa funtion ofthe angle of Figure 4.2 Voltage induced in the ac generator as a function of the angle of rotation rotation (Fig. 4.2). The waveshape depends upon the: shape of the N.S poles. We assume the poles were designed to generate the sinusoidal wave shown. Th speed, therefore each angle of rotation corresponds foaspecitic interval of time. Because the coil makes ‘one turn per second, the angle of 360° in Fig, 4.2 cor responds to an interval of one second. Consequently. We can also represent the induced vollage as a Func tion of time (Fig. 43), coil in our example revolves at unitorm Figure 4.3, Voltage induced as a function of ime 4.2 Direct-current generator IF the brushes in Fig. 4.1 gould be switched from one Sip cing to the other every time the polarity was hout to change. we woeld abun a voltage of eon stant polarity across the load, Brush x would always be positive and brush y negative, We ean obtain this result by using a commurator (Fig. 44). A comm {ator in its simplest form is composed of a slip fing that is cut in half, with each segment insulated fom the other as well as From the shaft, One segment is connected ty eoi-end A and the other to coil-end “The commutator revolves with the col and the volt age between the segments is picked up by two Sa tionary brushes x and y The voltage between brushes x and y pulsates but never ehanges polarity (Fig, 4.5), The alter- nating voltage in sho coil is rectified by the com mutator, which aets as a mechanical reversing switch Figure 4.4 Elementary de gonerator is simply an ac generator ‘equipped witn a mechanical rectifier called a commu: \I\ 780 270 360 cays = anole 3 0 Figure 4.5 The elementary de generator produces a pulsating de votage. DIRECT. CURRENT GENERATORS 13 Due to the constant polarity between the brashes. the current in the external load always flows in the ‘same direction, The machine represented in Fig. 4 is called a dirwer-current generator: or dynamo. 4.3 Difference between ac and de generators The clementary ac and de generators in Figs. 4.1 and 4.4 are essentially built the same way. In each case. coil rotates bevween the poles of a magnet and an ae voltage is induced in the coil. The mae chines only differ in the way the evils are connected. cireuit (Fig, 4,69: ae generatoryeany Sip rings (Fig 4.6h) while de generators requi ‘commutator (Fig. 4.63). We sometimes build sia machines which carry both slip rings and a commu: tator (Fig, 4.6c). Such machines can function si: ‘multaneously as ae and de generators 1 the extern 4.4 Improving the waveshape Returning to the de generator, we ean improve the pulsating de voltage by using four coils and four segments, as shown in Fig. 4.7. The result shape is given in Fig. 4.8, The vol but it never falls to zero itis much closer toa steady sill pulsates| de voltage. By increasing the number of coils and segments, we cam obtain a de voltage that is very smooth, Mavlern de generators produce voltages havin ripple of less than 5 percent. The coils are lod the slots ofa laminated ison eylinder. The coils and the eylinder constitute the armature of the machine. ‘The percent ripple is the ratio of the RMS value of the ae component of voltage to the de component, din {]_ |r - a] [ee o Figure 4.5 oy The tree armatures (a), (b), and (c) have identical windings. Depending upon how they are connected (to sp rings or a commutator), an ac or de voliage is obtained. 74 ELECTRICAL MACHINES AND TRANSFORMERS Figure 4.7 Schematic diagram of a de generator having 4 cols, and 4 commutator bars. See Fig. 4.9 90180270960 ae Figure 4.8 ‘The voltage between the brushes is mare unitorm than in Fig. 45 of Fig. 4.7. because we will be using similar drawings Iti important co understand the physical meani toexplain the behavior of de machines. The four cols inthe figure are identical to the coil shown in Fig. 4.1 AL ihe instant shown, coil A is not cutting any ux, and neither is coil C. The reason is that the coil sides of theve two coils are midway between the poles, On the other hand, coils B and D are cutting flux coming from the center of the N and S poles. Consequensy the voliaze tadaced in these coils is at its maximum possible value (20, say), That is albo the voltage across the brushes a this particular insta, Figure 4.9 “The actual physical construction of the generator shown in Fig. 47, The armature has 4 slot, 4 coils, land 4 commutator bars A schematic diagram such as Fig. 4.7 tells us where the coil sides of the individual coils ane lo- cated: between the poles, ender the poles. near the pole tips. and soon, But we must remember that the coil sides (a, as by, by: ete.) of each coil are act ally located gt 180° to each other and not side by side as Figure 4.7 seems to indicate ‘The actual construction of this armature is shown in Fig. 49, The four coils are placed in four slots. Each coil has two coil Sides, and so there are tsvo coil sides per slot. Thus, exch slot contains the conductors of two coils For reasons of symmetry, the coils ure wound so thar one coil side isa the bottom of a slot and the ‘other is at the top. For example. in Fig. 4.7 coil side aa isin the top of slot 1, while coil side as is im the bottom of slot 3. The coil connections to the com: ‘mutator segments are easy to follow in this simple armature. The reader should compare these connec tions with those in Fig, 49 10 verify that they are the same, Note also the actual position and schematic position of the brushes with respect to the poles, Fig, 4.10 shows the position of the coils when the armature has moved thrash 45°, The sides ay ay of coil A are now sweeping past pole tip | and pole tip 4. The sides of coil C are experiencing the same flux because they are jn the same slots as coil A, Consequently. the voltage e, induced in coil Ais rotation tip? Figure 4.10 Position of the coils when the armature of Fig. 4.9 has ‘ated through 45°, cevaclly the same as the voltage e, induced in coil Note. however, that coil A is moving downward. while coil C is moving upward. The polarities of ¢, and ¢, are, therefore, opposite as shown, The same reasoning leads us to conelude that ¢4, ane, are equal and opposite in polarity. This means thate, +e, + e+ €4 = Ota times, Consequently, no curent will flow in the cloved loop formed by the four coils. This is most fortunate, because any such, circulating current would produce FR losses ‘The voltage between the brushes is equal to €, + (ore, + ey) al the instant shown, It corresponds, ‘othe minimum voltage shown in Fig. 48. The armature winding we have just discussed is called a fap sinuing. It is the most common type of winding used indirect sseneratorsand motors, 45 Induced voltage Figures 4.11 and 4.11b show a more realist ar- mature having 12 coils and 12 slots instead of only 4.When the armature rotates the voltage E induced ia cach conductor depends upon the flux density which it cus. ‘This fet is based upon the equation 25) E= Bl fe Because the density in the aif gap varies from Point 0 point, the value of the induced voltage per coil depends upon its. instantaneous position, DIRECT-CURRENT GENERATORS 75 rotation neutral zone BY. coil A coil D Prysical construction of a de generator having 12 coils, 12 slots, and 12 commutator bars. Sh hay coll 8 Figure 4.11b Scnematic dagram of the armature and the voltages induced in the 12 cots, Consider, for example, the voltages induced in the armature when it occupies the position shown in Fig. 4.11. The conductors in slots and 7 ae ex- actly between the poles, where the flux density is zero. The voltage induced in the two coils lodged in slots | and 7 is, therefore, zero. On the other hand, the conductors in slots 4 and 10 are directly under the center of the poles, where the flux densi greatest, The voltage induced in the nwo coils 16 ELECTRICAL MACHINES AND TRANSFORMERS in these slots is, therefore, maximum, Finally. due to magnetic symmetry. the voltage in ‘duced in the coils loxlged in slots 3 and 9 isthe same as that induced in the coils Jouged in Sots Sand U1 Figure 4.11 shows the instantaneous Voltage in duced in each of the 12 coils of the armature, They are 0, 7, 18, and 20 V, respectively. Note that the brushes short-circuit the coils in which the voltage is momentarily zero. Taking polarities into account, we ean soe that the voltage between the brushes is (7 + 18 + 20 + I8 + 7) = 70 Vand brush x is positive with respect to brush y. This voltage remains essentially con: stant as the armature roles, because the auruber of coils between the brushes is always the same, ire spective of armature position Note that brush x in Fig. 4.11 straddles two {commutator segments that are connected 10 eoil A Consequently, the brush show-cireuits coi A. However, since the induced voltage in this coil is momentarily zero, no current will flow through the brush. The same remarks apply to brush y. Which momentarily shor-cireuits coil B. The brushes are sad to be in the neural position when they’ ate go sitioned on the commutator s© as to short-circuit those coils in whieh the induced voltage is momen- larity zero. That is the ease in Figs, 4. 1a and 4.1, I we were to shift the brush yoke by 30° (Fig, 4.12), the voltage between the brushes sould be: come (0+ 7+ 18 + 20+ 18) = 63 V. Thus, by shilling the brushes the ouput voltage decreases, Furthermore, inthis position, the brushes, continually short-circuit coils that generate 7 V, Large currents will flow in the short-circuited coils and brushes, and sparking will result, Thus, shifting the brushes olf the neutral position reduces the volt- re between the brushes and at the same time ses sparking, When sparking ovcurs, then is said 10 be poor commutation, 4.6 Neutral zones ‘Neutral zones are those places on the surtace of the armature where the flux density is zeto, When the _getterator operates a 0-load, the neutral zones are located exactly between the poles. No vol Figure 4.12 Moving the brushes off the neutral point reduces the ‘output voltage and produces sparking ‘duced ina coil that cuts through the neutral zone. We always try to set the brushes so they are in contact with coils that are momentarily in a neutral zone: 4.7 Value of the induced voltage The voltage induced in a de generator having a lap winding is given by the equation B= Zrbi60 an where £, = voltage between the brushes [V] Z = total number of eondyetors onthe armature n= speed of rotation teytnin{ «) = flux per pole |W] This important equation shows that for a given gen erator the voltage is directly proportional 1o the flax per pole and to the speed of rotation. The equation ‘only folds true ifthe brushes are on the neutral pos tion, Ifthe brushes are shifted off neutral, the effec isequivalemt to reducing the numberof eonduetorsZ, Example 4-1 —___ The armature of a 6-pole. 600 r/min generator, has) slots. Each enil bas tums and the Mux per pole is 0.04 Wb. Calculate the value of the induced voltage Solution Each turn corresponds to two conductors on the ar ‘mature, and 90 eoils are required 10 fill the 90 slots, The tol number of arm sre conductors is Z= Wecoils x 4 tums/evil % 2 conductossiurn 20 The speed is. = 600 r/min Consequently F,= Znbi6o 288 720 600 0.04/60 The voltage herween the brushes at no-load is there Fore 288 V. provided the brushes sie on neutral 48 Generator under load: the energy conversion process When a direct-current generator is under load, some undqental Mus andl current relationships take place that are diteedy related to the mechanice ‘conversion process. Consider for example, 2-pole generator that is diven counterclockwise while delivering current [10 a load (Fig. 4.13), Figure 4.13 ‘The energy corwersion process, The electromagnatic torque due to F must be balanced by the appiied me- chanical torque DIRECT-CURRENT GENERATORS 77 ‘The current delivered by the generator also lows through all the armature conductors. IF we could look inside the machine. we Weald discover hat current always flows in the same direetiva i thove conductors that are momentarily under a N pole. The same is true for conductors that are mo: imentaily under a pole. However, the currents un: der the N pole flow in the opposite direction 10 those under aS pole, Referring to Fig, 4.13. the ar ‘mature conductors under the $ pole carry currents that flow imo the page. away from the reader Conversely, the armature currents under the N pole flow out of the page. toward the reader, Because the conductors lie in a magnetic eld they are subjected to a force. according to Lorent2’s law (sections 2,22 and 2.23) If we examine the di- rection of current flow and the direetion of flux. we find that the individual forees F on the conductors alll yet clockwise, In effect, they produce « torque that acts opposite the direction in whicb th trator is being driven, To kes ‘we must exert a torque on the shisito overcome this, ‘opposing electromagnetic torque, The resulting me- chanical power is converted into eleettical power, which is delivered to the generator load. That is how the energy conversion process takes place. 4.9 Armature reaction Until now, we have assumed that the ony m motive force (mmf) acting ina de generator i that due to the field, However. the current Swing inthe arma ture coils also creates a powerflll maznetonwive fore that distorts and weakens the flux eomi poles. This distortion and field weakeniny rom the akes place in both motors and generators. The effect produced by the armature num is called asnatane reaction. To understand the impact of the armature ment ‘magnetic Vield ay shown in Fi 4.14. This field acts at right angles to the field produced hy the N.S poles The intensity of the armature Hux depends upon its ‘mnt, which in urn depends upon the current cated by the armature. Thus, contrary (othe Field flux. the armature flux is not constant ut varies with the Foad 78 ELECTRICAL MACHINES AND TRANSFORMERS Figure 4.14 Magnetic fiela produced by the current flowing in the armature conductors, We can immediately foresee a problem which the armature fux will produce, Fig. 4.14 shows that the Sux in the neutral zone is a longer zero and, COW sequently, voltage will be induced in the cols that are short-circuited by the brushes. As a result, severe Sparking may occur. The intensity of the sparkit Will depend upon the armature flax and kence «part the load current delivered by the generator: The second problem efeated by the armature mmf is that it distorts the flux produced by the poles. In effect, the combination of tbe armature ‘mmf and field mnt produces a magnetic tield whose shape is illustrated in Fig. 415. The neutral zones have shifted in the direction of rotation of the armature, This occurs in all de generators The flux distortion produces still another effect: the higher flux density in pole tips 2. 3 causes stu ration to set in, Consequently, the increase in flux. ‘under pole tips 2, 3 is less than the decrease in fu. under pole tips 1. 4, As a result, the total Nux pro- duced by the N.S poles is less than it was when the generator was running at no-load, This causes a cor. responding reduction in the induced voltage given bby By. 4.1, For fare machines. the decrease in flux may be as much as 10 percent Figure 4.15 ‘Armature reaction distorts the field produced by the N, S poles. I is important to note that the orientation of the armature fTux remains fixed in space: it does not ro- tate with the armature 4.10 Shifting the brushes to improve commutation Due to the shift fa the neutral zone when the gener ator is under load, we could move the brushes tore duce the sparking For generators the brushes ate shifted to the new neutral zone by moving thom in the digection ot 0: tation, For motors, the brushes are shifted against the direction of rotation, As soon as the brushes are moved, the commu. tion improves, meaning thew iv less sparking. However. if the load fluctuates. the armature mint rises and falls and so the neutral Zone shifts back and forth between the no-load and full-Joad positions We would therefore have to move the brushes hack and forth to-obcain sparkless commutation, This pro- cedure is not practical and other means are used to resolve the problem, For small de machines, how- ever the brushes are set in an intermediate position tw ensure reasonably good commutation a all loads. 4.11. Commutating poles To counter the effect of armature reaction in re-power de machines, we always place set of commutating poles between the main poles (Fig. 4.16). These narrow poles carry wind: ings that are connected in series with the armature. “The number of turns on the windi thatthe poles develop a magnetomotive force mm {equal and opposite 10 the magnetomotive force mmf, of the armature. As the Toad current varies the two magnetomotive forces rise and fall together, exactly bucking each other at all times, By nullify the armature mmf in this way, the flux in the rmediui- and lar space between the main poles is always zero and so wwe no longer have to shift the brushes. In practice ou ‘commutating ole Figure 4.16 Commutating poles produce an mmf, that opposes the mm, ofthe armature. Conting ples ave sometimes called tery DIRECT-CURRENT GENERATORS 79 the mmf of the commutating poles is made slightly greater than the armature mmf. This ereates a small flux in the neutral zone, which aids the commuta- tion process (see Section 4.28), Fig, 4,16 shows how the commutating poles of pole machine are connected, Clearly. the direc tion of the current flowing through the windings in dicates that the mmfof the eommutating poles acts ‘opposite to the mmf ofthe armature and, therefore, neutralizes its effect. However. the neutralization is restricted 10 the narrow brush zone where com= mutation takes place, The distorted Mux distribu tion under the main poles, unfortunately, cemains the same. 4.12 Separately excited generator Now that we have learned some basie fuets about de generators, we can study the various types and their properties. Thus. instead oF using permanent ‘magnets to create the magnetic field. we can use a pair of electromagnets, called field poles, as shown in Fig, 4.17. When the de field current in such & generator is supplied by an independent source (such as a storage buttery or another generator, called an exciter), the generator is said to be sepa: rately excited. Thus. in Fig. 4.17 the de source con: rected to terminals a and b causes an exciting cur- rent /, fo flow. It the armature is driven by a motor ‘or a diesel engine, a voltage , appears between brush terminals x and y. Figure 4.17 Separately excited 2-pole generator. The N, S field Poles are created by the current flowing inthe field windings. SO ELECTRICAL MACHINES AND TRANSFORMERS 4.13 No-load operation and saturation curve When a separately excited de generator runs at no. load (armature circuit open). changy ing current causes a corresponding change inthe iv duced voltuge. We now eunine the 9 between the N40, Field flux vs exciting current. Let us gradually raise the exciting current /. $0 that the mmf of the Field inereases, which increases the flux per pole Tt we plot as. function of J. we obtain the sara ration cwve of Fig, 4.18% This curve is obtained whether or not the generator is turn rated te Figure 4.188 Flux per pole versus exciting current When the exciting current is relatively small. the flay is small and the iron in the machine is un- Saturated, Very Tittle oomP és needed to establish ‘ie fTuy through the iton, with the result that the inmf deyeloped by the field coils is almost entirely ble to drive the fluy through the air gap, © the permeability of air is con flay increases in direct proportion to the exciting current. as shown by’ tbe near portion Oa of the st However, as we continue to raise the exciting ‘current. the jnon inthe field and the armature begins (o saturate, A farge increase in the mmf is now Re quired to produce a small increase in flux. as Showa, by portion be ofthe curve. The araciine is now said ter be saturated, Saturation of the iron begins to be Jn the so-called “knee” ab of cant. the important when we re the saturation eurve. How d voltage e saturation curve relate to the induced "If we drive the generator al constant speed, E,isdreetly proportional tothe fly «b, Consequently, by plotting &,, as a funetion of /,, we obtain a eure whose shape is idem! av he sat §.18a. The result is shown in Fig. 41s: its ealled the no-toudl saturation eurve of tb The rated voltage of a de litle above the knee of the curve, In Fig. 4.18b. for ‘example, the rated (ornominal) voltage is 120 V. By varying the exciting curren, We can vary the in- duced voltage as we please, Furthermore, by vers the current, the flux will reverse and so, 00 will the polarity of the induced vot Induced voltage vs speed. For « given exciting current. the induced volt pro- portion othe speed, exe a fotfows from Eq. 4.1 on curve of Fig, nerator, renerator is usual increases in dite If we reverse the direction of rotation, the pol ity of the induced voltage also reverses, However, if we reverse both the exciting current and the direc: tion of rotation. the polarity of the induced voltage remains the V | candstio v0 ee 0 | 60 Figure 4.18b Saturation curve of a de generator. 4.14 Shunt generator A shunt-excited generator is a machine whose shun-field winding is connected in parallel with the armature terminals. so that the generator ean be self-excited (Fig, 4.19). The principal advantage of this connection is that it eliminates the need for extemal source of excitation How is selfexcitation achieved? When a shunt erator is started up. a small volt is induced ia Figure 4.19 a. Seltexcited shunt generator » Schematic diagram of a shunt generator. A shunt let is one designed to be connected in shunt (ater- nate term fr parallel) with tne armature winding. the armature, due to the remanent flux in the poles This voltage produces a small exciting current J, in the shunt field, The resulin acts in the same direction as the remanent flux, eausin per pole to increase. The inereased flux ine: Which increases. which increases the fluy still mor which increases , even more, and so forth, This progressive buildup continues until E, reaches 4amaximum value determined by the field resistance ad the degree of saturation, See next section small 415 Controlling the voltage of a shunt generator Iriseasy to control the induved voltage of a shunt erator. We simply’ vary the excitin rt by means of a rheastat connected in series with the shunt Held (Fig. 4.20), DIRECT. CURRENT GENERATORS 81 Figure 4.20 CControling the generator voltage with afield rheostat A ‘heostat is a resistor with an adjustable slicing contact To understand how the outpet voltage var suppose that £, is 120 V when the movable contact Piisinthe center ofthe rheostat, Fe move the con tact toward extremity m, the resistance R, between points p and b diminishes, which causes the excit: ing current to increase, This increases the TIux and consequently, the induced voltage Ey. On the other hhand, if we move the contuet 1oward extremity 0, R, Increases, the exciting current diminishes, the fax diminishes, and so £., wil fll We can determine the #0-load value of E,, if we know the saturation curve of the generator and the total resistance R, of the shunt field circuit between points p and b. We draw a straight line correspond: ing to the slope of R, and superimpose it on the sat uration curve (Fig. 4.21), This dotted line passes through the origin, and the point where i imerseets the curve yields the induced voltage. For example. ithe shunt field has a resistance of 50 and the sheostat is set at extremity m, then R, ~ 50.0, The line corresponding to R, must pass through the coordinate point E = S0V.I-= 1A, This line intersects the saturation curve where the voltage is 150 V (Fig, 4.21), That is the maximum voltage the shunt generator ean produce, By changing the setting of the rheostat. the total resistance of the field circuit increases, causing E, tw decrease progsessively. For example. if R, is in cereayed (© 120 (the resistance fine cuts the salt ration curve ata voltage B,, of 120 V If we continue to nase R,, a critical value will be reached where the slope of the resistance line is 2 3A Figure 4.21 The no-load voltage depends upon the resistance of the shuntfild circut equal to that of the saturation curve in its unsaturated region, When this resistance is attained, the induced voltage suddenly drops to zero and will ret for any R, greater than this critical value. In Fig. 4.21 the cttical resistance corresponds to 200 2 4.16 Equivalent circuit We have seen that the armature winding contains a set of identical coils, all oF which possess acertain re sistance, The total armature resistance Ry is that which exist between the armature terminals when the machine is stationary. It is measured on the con mutator Surface between those segments that lie un- der the (+) and (=) brushes, The resistance is us ally very small, often less than one-hundredth of an ‘ohm, lis value depends mainly upon the power and voltage of the generator: To simplify the ceerator irouit. we can represent R, as i it were in series with ‘one of the brushes. I the machine has interpoles. the resistance of these windings is incladed in R, The equivalent circuit ofa generator is thus com. posed of a resistance Xin series with a volta (Fig, 4.22), The latter is the voltage induced in the 2 ELECTRICAL MACHINES AND TRANSFORMERS F 2 3 ©) Figure 4.22 Equivalent circuit of a de generator. revolving conductors, Terminals 1, 2 are the external armature terminals of the machine, and FF are the field winding terminals, Using this circuit, we will now study the more common types of direet-current generators and their behavior under load, 4.17 Separately excited generator under load Let us consider a separately excited generator that is driven at constant speed and whose field isexcited by battery (Fig. 4.23). The exciting curtent is constant ‘and so isthe resultant flux. The induced voltage E, is therefore fixed. When the machine operates at n0- load. terminal voltage E\3 is equal 10 the induced voltage £,, because the voltage drop in the armature resistanee is zere, However, if we connect a load across the armature (Fig. 4.23), the resulting Toad current J produces a voltage drop across resistance R,, Terminal voltage E, is now less than the induced voltage E,, As we increase the load, the terminal voltage diminishes progressively, as shown in Fig. 4.24. The graph of terminal voltage as a Tunetion of Toul current is called the foad curve of the generator. a ‘ Figure 4.23 ‘Separately excited generator under load. Figure 4.24 Load characteristic ofa separately excited generator In practice, the induced voltage E, also deere slightly with increasing load, because pole-tip satu- ration tends to decrease the field flux. Consequently, the terminal voltage > fally off more rapidly than can be attributed to armature resistance alone. 4.18 Shunt generator under load The terminal voltage of a selFexcited shunt generator Fall off mere sharply wit increasing load than that of separately excited generator, The reason is that the Feld current in a separately excited machine remains constant, whereas in a self citing current falls as the terminal vsltage drops. For selfexcited generator, the drop in voltage from no- load to full-load is about 15 percent of the faibload voltage, whereas for a separately excited generator it isusually less than 10 percent, The voltage regulation issiid tobe 15% and 10%. respectively. fed generator the ex- 4.19 Compound generator The compound generator was developed to prevent minal voltage of a de generator from de- g with ineveasing load. Thus, although we can usually tolerate a reasonable drop in terminal as the load increases. this has a serious ef cuits. For example, the distribu tion system of a ship supplies power to both de mi chinery and incandescent lamps. The current delivered by the reyponse to the varyi enerator fluctuates continually, in 1g loads. These current varia DIRECT-CURRENT GENERATORS 3 tions produce corresponding chang. tor terminal voltage, causing the lights to tlicker: Compound generators eliminate this problem, ‘A compound generator (Fig. 4.25a) is similar to.a shunt generator. except that it has field coils connected in series with the armature These series field coils are composed of a few turns of heavy wire, big enough to carry the armature cur- rent, The total resistance of the series coils is, there: fore, small. Figure 4.25 showing the shunt and series field connections. When the generator runs at no-load, the current inthe series coils is zero, The shunt coils, however, carry exciting current £, which produces the field flux, just as in a standard self-excited shunt gener- ator. As the generator is loaded, the terminal volt- age tends to drop, but load current J, now flows, through the series field coils. The mmf developed by these coils acts inthe same direction as the mm Oo the shunt field. Consequently, the field flux un= der load rises above its original no-load value, which raises the value of £,. Ifthe series coils are wned, the terminal voltage temains ant from no-load to full-load. The is a schematic diagram eld stunt fea TH oi shunt cl 4 Figure 4.25 a. Compound generator under load. ». Schematic diagram. Sd ELECTRICAL MACHINES AND TRANSFORMERS rise im the induced voltage compensates for the ar mature IR drop. Insome caves we have to compensate not only for the armature voltage drop. but also forthe IR drop in the fseder line between the generator and the Kad. Tie yenerator manufacturer then adds on tra turns on the series winding so that the terminal voltage increases as the load cutrent rises, Such ma. chines are called aver-compounud generators. WF the ortwoex- ‘compounding is too strong. a low resistance can be placed! in parallel with the series field, This reduces the current in the seris field and hs the same effect as reducing the number of turns. For example. if the value of the diverter resistance is equal to that of the series Held, the current in the laters reduced by hal 4,20 Differential compound generator Ina differential compound generator the mont ofthe series field acts opposite tothe shunt field. As are- sult, the terminal voltage falls drastially with in creasing loud, We car make such a generator by simply reversing the series Field ofa standard com pound generator. Diffeyential compound generators were formerly used in de are welders, because they tended to init the shor -circuit curren and to tab lize the ane during the welding process. The vokage regulation of the differential com pound generator in Fig, 4.26 is (no-load ~full- loadull-toad = (100 —70y70 = 42.9% 100 eompouna on arte saon zie -itferenti compound » oO Figure 4.26 ‘Typical load characteristics of dc generators. 4.21 Load characteristics ‘The load characteristics of some shunt and com: pound generators are given in Fig. 4.26. The vole age of an over-compound generator inereases by 10 percent when full-load is applied. whereas that of a fat-compound generator remains constant. On the other hand, the full-load voltage of a shunt gener Loris 15 percent below its no-load value, while that of a differential-compound generator is 30 percent ower 4.22 Generator specifications ‘The nameplate of a generator indicates the power, voltage, speed, and other details about tbe mackie These the values guaranteed by the manufacturer. For exam ple, the following information is punched on the nameplate of a 100 kW generator 100 KW. Speed ings, or nominal characteristics, 3 Power 1200 r/min Voliage 250 V Type Compound Exciting curren 20, A Class. B Temperature rise 50°C ‘These specifications tell us that the machine ean Ueliver, continuously, a power of 100 KW ata volt- of 250 V, without exceeding a temperature rise ‘of 50°C. It can therefore supply 3 load current of 100000/250 = 400 A. It possesses a series winding. ‘and the current in the shunt field is 20. In practice the terminal voltage is adjusted 10 a value close 1 its rating of 250 V. We may draw any amount of ower from the {ceed 100 kW and the current is less than 400A. The class B designation refers (0 the class of insulation used in the machine. nerator, as long as it does not ex- CONSTRUCTION OF DIRECT-CURRENT GENERATORS We have described the basic features and properties of ditect-curtent generators. We now look at the ‘mechanical construction of these machines, diect- ing our atention o the field, the armature. the com mutator, al the brushes. 4.23 Field The field produces the magnetic Mux in the ma- chine. It is basically a stationary electromagnet composed of a set of salient poles bolted to the in side of a circular frame (Figs, 4.27 and 4.28). Field ceils. mounted on the poles, carry the de exciting ccurent. The frame is usually made of solid cast Steel. wheteas the pole pieces are composed of stacked iron laminations, In some generators the Tux is ereated by permanent magnets. Inourdiscussions so far we have considered only 2-pole generators. However, in practice a de gener tor oF motor may have 2, 4, 6, or as many as 24 poles. The number of poles depends upon the phys- Figure 4.27 ‘Gross section of a 2-pole generator. Figure 4.28 Cutaway view of a 4-pole shunt generator Ithas 3 brushes per brush set DIRECT-CURREST GENERATORS 85 Figure 4.29 Adjacent poles of multiple generators have apposite magnetic polarities. ‘eal size of the machine: the bigger itis. the more poles it will have, By using a multipole design, we ‘can reduce the dimensions and cost of Tange mu chines, and also improve their performance The field coils of a multipole mackine are con: rected together So that aujacent poles have oppo agnetic polarities (Fig. 4.29). The shunt field coils are composed of several hundred turns of wire carrying a relatively small current, The coits ‘are insulated from the pole pieces to prevent short- site mi circuits The mmf developed by the coils produces a ‘magnetic flux that passes through the pole pieces. the frame, the armature, and the air gap. The air gap. isthe short space between the armature and the pieces. It ranges from about 1.5 t0 $ mmas the ‘erator rating increases from | kW to 100 kW. Because the armature and field are composed of magnetic materials having excellent permeability, most of the mmf produced by the field is used 1 rive the flu aeross the air gap. Consequently. by reducing its length, we can diminish the size of the shunt field coils. However. the air gap cannot be ‘made too short otherwise the armature reaction Fect becomes too great If the generator has « series field, the coils are wound 01 top of the shuat-tield coils. The condy tor size must be large enough so that the winding does not overheat when it carries the fulload eur rent of the genes 86 ELECTRICAL MACHINES AND TRANSFORMERS 4.24 Armature The armature is the rotating pan of a de generator. I sls oF a commutator, an ito» core, and a set of coils (Fig. 4.30), The armature is Keyed to a shatl and revolves between the field poles. The iron core is ‘composed of slotted, iron laminations that are stacked to form a solid cylindrical core, The laminations are individually coated with an insulating fi so that they do not come in electrical contact with each other Asa result, eddy-current losses are reduced, The slots are I mature conductor, The armature conductors carry the load curreat delivered by the generator. They are insulated from the iron core by several layers of paper or mica and are firmly held in phice by fiber slo sticks. Ifthe ar mature current is befow 10 A. round wire is used: but for currents exceeding 20 A. rectangular con- ductors are preferred because they make better use of the available slot space, The lamination of a sama armas A cross see tion view of the slot ofa large armature is shown in Fig, 4.32. el up to provide the space needed to insert the is shown in Fig. 4.31 Figure 4.30 ‘Armature of a de generator showing the commutator, Stacked laminations, slots, and shatt, (Courtesy of General Elecric Company, USA) Figure 4.31 ‘Armature lamination with tapered slots Z 4 Figure 4.32 ‘Cross-section ofa slot containing 4 conductors. 4.25 Commutator and brushes ‘The commutator is composed of an assembly of ts pered copper seaments insulated from each other by mica sheets. and mounted on the shaft of the ma chine (Fig. 4.33), The armature conductors are cos nected to the commutator ip tanner we will ex plain in Section 4.26, Great care is taken in building the commutator because any eccentricity will eause the brushes 10 bounce. uunaceeptable sparking. The sparks burn the brushes and overheat and carbonize the commutator product mica segment Figure 4.39, Commutator of a de machine. A 2pole generator has two brushes fixed dia- inetrically opposite o each other (Fig. 4.342). They slide on the commutator and ensure good electrical ‘contact berween the revolving armature and the sa tionary external Toad “Multipole machines possess as many brush Sets as they have poles. The brush set, in turn, are composed of one or more brushes, depending upon the current that has to be carried. In Fig, 4.35e, for example, wo bushes mounted side-by-side make up the brush set ‘The brush sets are spaced at equal intervals around the ‘commutator. They are supported by a movable brush Figure 4.94 a. Brushes of a 2-polo gonerator. . Brushes and connections of a 6-pole generator. DIRECT. CURRENT GENERATORS 87 yoke that permits the entre brush assembly 10 be ro tated through an angle and then locked in the neutral Poxition. In going around the commutator, the su ‘cessive brush sels have positive and negative po lies. Brushes having the same polity are connected Together and the leads are brought out to one positive and one negative terminal (Fig. 4.34b) ‘The brushes are made of carbon because it has good electrical conductivity and its softness does not score the commutator. To improve the conduc tivity, a small amount oF copperis sometimes mixed ‘with the carbon. The brush pressure is Set by means ‘of adjustable springs. I'the pressure is too great, the Friction produces excessive heating of the commu tator and brushes: on the other hand. iF it is too ‘weak, the imperfect contact may produce sparking, @ Figure 4.35 ‘a. Caroon brush and ulrattexible copper lead ». Brush holder and spring to exert pressure. ©. Brush set composed of two brushes, mounted on rocker arm, (Courtesy of General Elects Company. USA) 88 ELECTRICAL MACHINES AND TRANSFORMERS ‘The pressure is usually about 1S KPic C= 2 thin’), and the peraissibfe current density is approxi- mately 10 Afem (= 65 Afi). Thus, tt ical brush having a eross section of 3em X Lem (= 12in x ‘0-4 in) exerts a pressure of 4.5 N (|b) and ean ceary a cient of about 30 A Wt 4.36 shows the construction af a modern 4epole de generator. In onder 10 appreciate the progress that has been mule, Fig. 4.37 showsa gen ‘erator tht Was butt in 188. Figure 4.36 ‘Sectional view of a 100 KW, 250 V, 1750 r/min 4-pole de generator, (Courtesy of General Electric Company, USA) 4.26 Details of a multipole generator In onder to get a better understanding of multipole generators, let us examine the construction of a 12 pole machine, Fig. 4.38a is the schematic diagram ‘of such a machine having 72 slots on the armature, 72 segments on the commutator. ane 72 coils. The armature has a lap winding. and the reader shout note how similar itis <9 she schematic diagram of a pate machine (ig 4116) Coils Aand © ae mo Figure 4.37 This drect-curent Thompson generator was fist ine ‘saled in 1889 to light the streets of Montreal It el- ‘eda current of 250 A ata veliage of 110 Other prop Cees of this pioneering machine include the folowing: Speed 1300 r/min Total weight 2390) Armature diameter 282 mm, Stator internal diameter 330 mm Number of commutator bars 16 ‘Armature conductor size a4 Shunt field conductor size #4 ‘A modern generator having the same power and speed weighs 7 times less and occupies only 1/3 the foor space cntarily in the neutral zone, while coil Bis euting the Mux coming from the centera the poles. “The coil width (known as coil pitch) ig such that the col ides cut the flux coming from adjacent N. S pales. Thus. the coil sides of coil B lie under the center of pole 2 and the center of pole 3. Similarly. the coil sides of eoil A are in the tween poles 1, 2 and poles 2, 3 cal zones be= DIRECE-CURRENT GENERATORS. 89) The vollage generated between brushes xan y is equal 10 the sum ofthe volt rate by ie Five coils connected to commutator segments. 1-2, 23, 3-4. 4-5. and 5-6. The voitages between the other brush sets are similarly generated by five cols The (+) brush sets are connected together Form the (+) terminal, The (—) brush sets are simi- larly conneeted to form the (=) terminal. These connections are not shown on the diagram, For sim= ilar reasons of larity. we do not shove the interpoles, that ure placed between the N, S poles Fig, 4.38b gives a detailed view of the armature coils lying between brushes x and y. Only the three coils A, B, and Care shown sos not tocomplicate the diagram, Coil Ahas ts col sides in Slots Land 7. whi those o coil B are in slots 4 and 10. Furthermore. evil Ais connected 10 commutator segments 72 and 1. while coil B is connected to segments $ and 4 In the position show, the eoib-vides of coil A are inthe neutral zone between the pos. Consequert! ‘no voltage is induced in coil A. On the other hand, the coil sides of B are dirgetly under the Nand S poles. The voltage in coil B is praximum at this m0- ment. Consequently, the voltage between adjacent commutator segments 3 and 4 is maximum, ‘The voltage in coil C isalso zero because its evil sides ure sweeping seross the neutral zone. Note that the positive and negative brushes each short= circuit coils having zero induced voltage Example 4-2 —_ ‘The generator in Fig, 4.38 generates 240 V hewween adjacent brushes and delivers a cusrent of 2400 A, tw the load, Caleukae a The current delivered per brush set b. The current flowing i each coil . The average voltage induced per coil Solution ‘a. Acurrent of 2400 A flows out of the (+) terminal tnd back into the (—) terminal of the generator, ‘There are 12 brush sets 6 positive and 6 negative. ‘The current per brush set is 1= 2400/6 = 400.8 90 ELECTRICAL MACHINES AND TRANSFORMERS Figure 4.384 ‘Schematic diagram of a 12:pole, 72coll de generator. Figure 4.380 Closeup view of the armature coils between adjacent brushes. »b, Each positive brush set gathers current from the coils to the right and to the left of the brush, Consequently, the current in each coil is 40072 = 200. c. There are sia coils between adjacent brush sets, The average voltage per coil is Eggo = 20M = 40V 4.27 The ideal commutation process ‘When a generac is under load, the individual coils “onthe armature carry one-half the load current curied boy one brush. The currents flowing in the armature windings nest (© & positive brush are shown in Fig. 4.39a, Note that the currents in the coils flow toward the brush, coming both from the right and the let. I the lol current is 80, the coils all carry 40 A. If the commutator segments are moving from Fight to left, the coils on the right-hand side of the brush will soon be on the leti-hand side. This means that the current in these coils must reverse, The 1e- ‘versal fakes place during the millisecond interval that coil takes to move from one end of the brush 10 the tater. The process whereby the current changes di- rection i this brief interval is called commutation Tounderstand how commutation takes place. we refer to Figs. 4.39a to 4,396. In Fig. 4.394 the brush is in the middle of sey ment I. and the 40 A from the coils on the right and the left of the brush unite t give the 80 A output. ‘The contact resistance between the segment and brush proxluces a voltage drop of about I V. In Fig. 4.39b the commutator has moved a and 25 percent of the brush surface is now in contact with segment 2. while 75 per cent is in contact with segment 1. Owing to the the conductivity between the brush and commutator is proportional to the con- tact area, The area in contact with segment 2 is only one-fourth of the total contact area, and so the current from segment 2 is only one-fourth of the etal current, namely 0.25 x 80 = 20.A. By short distang contact resistans DIRECE-CURRENT GENERATORS 91 -! te) Figure 4.39 Commutation of the current in coil 1. Inductive effects are neglected and current reversal s caused by the brush contact resistance 92, ELECTRICAL MACHINES AND TRANSFORMERS the same token, the current from segment I to the brush is 0.75 «80 = 604. we now apply Kirehhol’s current law, we dis ceover that the current flowing in eoil L must be 20. Thus, by coming in contact with the brush, the eur= rent in this coil has dropped trom 40 A 10 20.4, In Fig, 4.39 the commutator has moved a file cher, snd the brush area in contact with segments and 2 is now the sume, Consequently the con ities ane the Sime sand So the currents are equal, This ‘means that the current in coil 1 iy zero a this instant In Fig, 4.39d the commutator has moved still far- ther to the elt. Segment 2 is now in contaet with 75 percent of the brush, and so the currents divide ccondingly: 60 A from segment 2.and 20 A from seg. ment I. Applying Kirehhot's current law, we find that the current in coil [is again 20 A, but it Flows in the opposite direction to what it did before! We ‘ean -pew understand how the brush contact resis- tance forees a progressive reversal oF the current as the segments slide over the brush. In Fig. 4.39 the current reversal in oil | iscome plete and the current in coil 2 is about to be reversed Inthis ideal commutation process. itis important to note that the current density (amperes per square centimeter) remains the same at every point across the brush face. Thus, the heat produced by the con: uct resistance is. spread uniformally aeross the brush surface, Unfortunately, such ideal commuta: tion is not possible in practical machines, and we ‘now investigate the reason why, 4.28 The practical commutation process ‘The problem with commut in a very short time: consequently. the current can not reverse as quickly as it should, The reason is that the armature coils have induct Mrongly opposes a rapid cha jon is that it takes place Suppose. for example, that the commutator in Fig, 4.39 has 72 bars and tha the armature turns at {600 shmin, One revolution is, therefore, completed in 1/10 of a second andl during this short period 72 commutator bars sweep past the brush. Thus, the time avaliable tore VIO x W72 = 1/7208 0r 1.39 ms! The voltage induced by sel-induetion is given by crse the current in coil Lis only LAWS 4. = induced voltage [VI L. = inductance of the coil [H] AMAA = rate of change of everent [A/S] IW coil | has an inductance of, say. 100 HL the duced voltage is © = LMS 100 x 10% x | +40 — 139 107% 400) 5 Ic is the presence of this induced voltay able 10 L) that opposes the change in current. Figs. 4.404 to 4.40¢ illustrate the new currents that How in coil | when the seli-inductance of the coil is considered. We have assumed plausible val tues for these currents in order (0 determine the me Sulting current flows in the brush, The currents, should be compared with those in Fig. 4.39. In Fig, 4.40 the brush is in the middle of seg- iment 1, and the currents in the coils are neither in- creasing or decreasing. AS a result, the col tance does not come into play. In Fig. 4.40b the current in coil | is changing due to the contact resistanee effect, However, the in- duced voltage ¢ prevents the current from dropping to its ideal value of 20 A, Suppose the coil current is 35 A. From Kirchhott's current lu, the currents flowing from segments 1 and 2 into the brush are then respectively 75 A and 5 A. instead of 60) A and 20 A, Note thatthe current density is no longer uni form over the brush face, The density is low wh the brush touches segment 2, and high where it touches segment | In Fig, 440¢ the brush is momentarily symmete- cally placed as regards segments | and 2, But the eur- ‘ot fallen to zero, and is still say e (attribute rent in coil (b) Figure 4.40 (Commutation of the current in col 1. The coll induc tance opposes the reversal of current DIRECT-CURRENT GENERATORS 98 30. As a resull the current in segment 1 is 70.4 while that in segment 2isonly 0A. Thecurrent den= Sity or the left-hand side ofthe brush is, therefore, 7 times greater than on the right-hand side. The left hhand side of the brush will vend 1o overheat, In Fig. 4.4041 segment I has moved beyond the midpoint oF the brush and the current in evil 1 has. sill not reversed. Assuming it has a value of 20 A, the current Mowing from segment 1 to the brush is snow 601A. despite the faet that the contael area is getting very small. The resulting high current den: sity causes the brush to overheat atthe tip. Because 720 coils are being ccanmutated every second. this ‘overheating raises the brush tip to the incandescent point and serious sparking will result In designing de motors and generators. every effort is made to reduce the self-induetance of the coils. One of the most effective ways is to reduce the number of turns per coil, But for a given out put voltage. this means that the number of coils must be increased. And more coils implies more commutator bars. Thus, in practice. direct-eurrent generators have a large number of coils and com- ‘mutator bars — not so much co reduce the ripple in the output voltage but to overcome the problem of commutation, Another important fxetor in aiding commutation is that the mmf of the commutating poles is always, made slightly greater han the sarmature-mmf Therefore. small Mux is ereated in the neutral zone, As the coil side undergoing commutation sweeps through this flux. a voltage is induced in the coil which opposes the voltage due to the self Inductance of the coil Inaddition to these me the brush is carefully chosen, It alfeets the brush voltage drop, whieh can vary trom 0.2 V toas much as 1.5 V. This drop occurs between the surface of the brush and the commutator surface, large brush drop helps commutation, but unfortunately it i creaves the losses. As a result the commutator and brushes become hovter and the efficiency of the sures, the composition of 94 ELECTRICAL MACHINES AND TRANSFORMERS Questions and Problems Practical level 4-1 Sketch the main components of a de zener- 4-2 Why are the brushes of a de machine al \ways placed at the neutral points? 4-3. Describe the construction of a commutator 4-4 How is the induced voltage ofa separately excited de generator affected if a the speed increases? bs. the exciting eurent is redaced? 45 How do we adjust the voltage of 2 shunt generator? 4-6 The terminal voltage of a shunt generator decreases with increasing load, Explain 4.7 Explain why the output voltage of'an overs ‘compound generator ineveases as the load 4-8 Explain theeiferone between shunt, compevind, an ferential compos by as wnelectrical properties eras Intermediate level 49. Aseparate excited de generator twrning at 1.400 r/min produces an induced voltage of 127 V. The armature resistance is 2.0 and the machine delivers a current of 12.4. Caleutare ‘8, The terminal voltage [V] bb The heat dissipated inthe armature (W] The braking torque exerted by the armature [Nm] A separately excited de generator pro- duces a no-load voltage of 115 V. What happens if 4. The speed is increased by 20 perent?” 1 The direction of rotation is reversed? ©. Theevet 410 1 currents increased hy 10 percent? ‘d. The polarity of the field is reversed? Each pole of 100 kW, 250 V flat-compound generator has a shut field of 2000 turns and series field of 7 tues, Ifthe oral shunt-tiekd resistance is 10002 caleulate the mint when the machine operates at rated voltag a. ACno-load D, Acfull-load Fig, 4.18 shows the no-oud saturation curve of a separately exeited de generator ‘when it revolves at 1500 r/min, Calculate the exciting current needed to 120 V at 1330 sien, Referring to Fig. 4.10. the induced voltage in coil D is momentarily 18 V. in the posi- tion shown, Calculate the voltages induced in coils A. B, and Cat the same instant nerate 413 4-14 Referring 10 Fig. 4.11, calculate the volt age induced in coil A when the armature has rotated by 90°; by 120° Brush x is positive with respect to brush y in Fig, 4.11b, Show the polarity of each of the 12 coils, Does the polarity reverse when ‘agoil turns through 180°? The generator of Fig, 4.38 revolves at 960. ut Calculate the no-load armature voltage if cach armature coil has 6 turns, 417 a ls 416 and the Mux per pole is 20 mWh, How many brush ses are need! fr the gen erator in Fig. 4.38? bi, I the machine delivers a otal load eurrent of 18041. calculate the current Flowing in each amature cil Advanced level 4-18 The voltage between brushes x and y is 240 V in the generator shows in Fig. 4.38. Why can we say that the voltage between segments 3 and 4 rnst be greater than 4ov? Referring to Fig. 4.10, determine the polar- ity of Fy, when the armature turns counter clockwise, 419 4-20 a, In Fig. 4.38 determine the potsity of Ex be {sveen commutator segments 3 and know ing that the stmature is turning clockwise AL the same instant, what is the pokaity of Segment 35 with respect 1 segment M? 421 The armature shown in Fig, 5.4 (Chapter 5) has 81 slots, and the commutator has 243 segments. It will be wound to give a 6-pole Jap winding having I urn per coil. the flux per field pole is 30 mWb, calculate the followi 1. The induced vol 1200 sti aa speed of flux density per poe fe. The time needed to reverse the eurrent in cach armature coil. Knowing that the brushes fre 15 mm wide and thatthe diameter ofthe ‘commutator is 450 mm, .\.200.W, 120 V, 1800 r/min de generator hhas 75 commutator bars, The brush width is such as to cover 3 commutator segments Show that the duration of the commutation process is equal to 1.33 ms, A d-pole 250 kW, 7S0V de generator has a lap winding on the armature. 422 423 Caleulave a. The fil-load curtent of the generator 1, The eurvent carried by the armature coils DIRECT-CURRENT GENERATORS 9S Industrial Application 4.24 ‘A240 kW, 500 V 1750 r/min separately excited de generator has an overall effi ciency of 94%. The shunt field resistance is 60 ohms and the rated current is 5 A. ‘The FR loss in the armature is 0.023 pu, Catewlare 4:26 ‘a, The rated armature current The total losses inthe machine The FR lowes in the armature ‘The generator in Problem 4-24 weighs 2600 th. Calensate the omtpat in watts per kilogram, In Problem 4.24 calculate the tor quired o drive the generator at 1750 r/min, (The shunt field is powered by a separate source.) A d-pole de generator delivers a current of 218 A. The average brush voltage drop on teach of the four brush sets is found 6 be (046 V. Caleulate the (otal brush Toss in the ‘machine, neglecting friction loss. CHAPTER 5 Direct-Current Motors 5.0 Introduction Dtitsicieagenduntrsinling fg Jerators, we ean begin our study of de motors, motors. transform ele 2 into mechanical enersy. They drive deviees stich as Fhoists fans, pumps. calendars, punch-presses. and as These devices may have a definite tonque-speed churaeterisie (such asa pump or fan) ora highly va able one (sich asa hoist or automobile). The torque speed characteristic of the motor must be adapted t0 the type of the lol it has to drive, and chis require ment has sven sve a ree bxsie types of motors Direet-curre 1. Shunt motors 2. Series motors. 3. Compound motors Direct-current motors are seldom used in ordinary industrial applications because all electric utility sys ems fumish altering current. However. for spe ial applications such as in steel mill mines, and electric trains. itis sometimes advant form the alternating current into direct current in der to use de motors. The reason is that the torgue= speed characteristics of Ge moseys ca be varied over wide range white retaining high elfivieney \eous to teas: 96 ‘Today, this general statement can be challenged because the availability of sophisticated electronic ives as made it possible to use alternatin motors for variable speed applications. Nevertheless there are millions of de motors sfill in service and thousands more are being proxluced every year ‘current 5.1 Counter-electromotive force (cemf) Direct-current motors are built the same _zenerators are; consequently, de machine ean op: fore either as motor oF as a1 generator, To ilus- ‘ate, consider a de generator in which the armature initially at vest, is connected to a de source F, by means of a sviteh (Fig. 5.1). The armature has a re- sistance R, and the magnetic Field is created by a set, Of permanent mi AAs soon as the switeh is closed. a large current lows in the armature because its resistance is ve low. The individus diately subjected @ a force because th mersed in the magnetic field ereated by the perm nent magnets. These Forces add up 10 produ ing the armature to rotate, armature condeceors ae i powerful torque, ct Figure 5.1 Stating a de motor across the line (On the other hand, as soon as the armature be _gns juin, a second phenouenon fakes pice: the generator effect. We know that a voltage is dluved in the armature conductors as soon as they ‘cuts magnetic Field (Fig. 5.2). This is always try no maarer shat causes the rotation. ‘The value and Polarity ofthe induced voltage are the same as those ‘obiained when the machine operates us a generator The inxluced voltage E, is therefore proportional to the speed of rotation 1 of the motor and to the flux «per pole, as previously given by Ey. 4.1 = Zibi60 ca Asinthe ease oF generator. Zia constant that de- penuls upon the number of turns on the armature and the ype of winding. For lap windings Z is equal 10 the number of armature conductors, Inthe ease of a motor. the induced voltage E,, is called counter-electromotive force (cent) because ity polarity always acts against the source voltag E. Itacts against the voltage in the sense that the net voltage aeting in the series circuit of Fig. 5.2 is equal to 1, — B,) volts and not (E, + B,) volts, Figure 5.2 CCounter-electromotive force (cemf) in a de motor DIRECE-CURRENT MOTORS 99 5.2 Acceleration of the motor The net voltage scting inthe armature circuit in Fig. 5.2is(E, ~ £,) volts. The resulting armature current Fis limited only by the armature res (EER GD Kee Ran so When the motor is at rest, the induced voltage (0, und so the starting eurres T= ER The starting curvent may be 20-10 30 times xsreater than the nominal full-load current of the mo- tor. In practice, this would cause 1 ‘oF the circuit-breakers to trip. However if they are absent, the large forces uet ductors produce a powerful starting toryue and a ‘consequent rapid acceleration of the armature. As the speed increases, the counteremt &, in- creases. with the result that the value of (E, — E,) diminishes. Ie follows from Eg. 5.1 that the armature current / drops progressively as the speed increases Although the arn motor continues to aecelerate until reaches a de inite, maximum speed. At no-load this speed pro: duces a counter-emtE, slightly lessthamn the source voltage En effect. iE, were equal to Ey the net voltaze (E, ~ £,) would become zero and 50. 0. ‘would the current. The driving forces would cease to.act on the armature conductors, aid the mec ‘cal drag imposed by the fan and the bearings would immediately cause the motor to slow down, As the et voltage (F, = F,,) nereases rent J. The speed will cease 10 Fuses 10 blow speed decreases the and so does the fall as soon as the torque developed by the armature current is equal to the Toad torque. Thus, when a motor runs at no-load, the counter-em must be slightly less than £,, so a 10 enables small current 10 flow. sulfieient to proguce the required lorque. Example 5-1 The armature of permanent-magnet de generator has e oF SOV aa resistance of 12 and generates a volts the speed is 500 r/min. Ith nected to source oF 150 V, calcutate the follow aa. The starting current 98 ELECTRICAL MACHINES AND TRANSFORMERS 50a to00riminx []10 GE il tsov 1 | ‘1 Figure 5.3, See Example §.1 b. The count motor runs at 1000 rimin, At 1460 r/min, ©. The armature current at 1000 rlmin, At 1460 vimin Solution a. Atthe moment of Sar-up, the armature iy Sta tionary, so &, = 0'V (Fig. 5.30). The starting cur- rent is limited only by the armature resistance: T= EIR = 150 V/A OQ= 150A b, Because the zenerator voltage is SO V at S00 slmin. the cemf of the motor will be 100 V at 1000 rimin and 146 V at 1460 r/min, . The net voltage in the arinature circuit at 1000 lin is 150 ~ 100 = 50V ‘The correspond r= ie armature current is EIR = SO/1 = SOA cFig. 5.30) When the motor speed reaches 1460 r/min, the ‘com will be 146 V. almost equal to the source volt age. Under these conditions. the armature current is only E,= ER = 150 ~ 146y11 =40 andthe corresponding motor sinaller than befire (Fig. 5.3e), tonque is much w 5.3 Mechanical power and torque ‘The power and torque of a de motor are two of its ‘most important properties. We now derive two sim- ple equations that enable us to ealeutate them. According to Bq, 4.1 the cemf induced in a lap- wound armature is given by Znsbi60 ap Referring to Fig. 5.2, the electrical power P, sup- plied to the armature is equal to the supply voltage multiplied by dhe armature current P= BL (52) However, £ is equal (© the sum of drop in the armature: E.= E+ plus the JR (53) It follows that PL Ed 64) The PR term represents heat dissipated in the ar= ‘mature, but the very important term E41 is the electrical power that is converted into mechanical power. The mechanical power of the motor is therefore exactly equal to the product of the cemf multiplied by the armature current PoEd (35) where P= mechanical power developed by the motor [W] induced voltage in the armature (cemf) vl 1 total current supplied to the ‘Turning our attention to torque T; we know that the mechanical power P is given by the expression mature [A] P= nTI9SS Gs) where 1 is the speed of rotation. Combining Eqs. 3.5, 4.1. and 5.5, we obtain nT9SS = Ed = Zn lies and so Z016.28 The torque developed by a lap-wound motor is therefore given hy the expression T= 2116.28 6.6) Figure 5.4 DIRECECURRENT MOTORS 99 where 1 torque [N-ma} = number of conductors on the armature ® = effective flux per pole [Wb|* 1 = armature current [A] 6.28 = constant to take eare of units exact value = 271 Eq, 5.6 shows that we ca raise the torque of a motor either by raising the armature current of by raising the flux ereated by the poles. Example 5-2 ‘The following details are given on a 30 np). 250 V, 1200 r/min de motor (see Figs. 5.4 amd 53) armature coils 243 turns per coil H type of winding lap armatare slots sl ‘commutator segments 23 field poles 6 diameter of armature 559 mm axial length of armature 235 mm sive Hs nse by P= 60 Zn Bare armature and commutator of a ds motor rated 225 kW, 250 V, 1200 rimin. The armature core has a diameter 0559 mm and an axial length of 235 mm. Itis composed of 400 stacked laminations 0.56 mm thick. The armature has 81 slots and the commutator has 243 bars. (H. Foberge) 100. ELECTRICAL MACHINES AND TRANSFORMERS @ (b) Figure 5.5 @ ‘a, Armature of Fig. 5.4 in the process of being wouné: col-forming machine gives the colls the desived shape. . One of the 81 coils ready to be placed in the slots ‘c. Connecting the coll ends to the commutator bars. 3. Commutator connections ready tor brazing. (H. Raberge) Catentate a. The rated armature current b. The number of conductors per slot The flux per pole Soluion a. We ean assume that the induced voltage E, is nearly equal to the applied voltage (250 V). ‘The rated armature eurrent is 1= PIE, = 900 = 225 000/250 b, Each coil is made up of 2 conductors, so alto: gether there are 243 X 2 = 486 conductors on the armature Conductors per slot = 486/81 = 6 Coil sides per slot = 6 e motor tonyue is T= 9.55 Pin 55 x 2) 1791 Nm 09071200, ‘The flux per pole is = 628 112 (6.28 x 1790/1486 x 900) 25.7 mW 5.4 Speed of rotation When ade motor drives a load between no-load and Full-load, the 7 drop due to armature resistance is always small compared to the supply voltage Ey ‘This means that the counter-emt E,, is very nearly ‘equal to Ey On the other hand, we have already seen that E, may be expressed by the equation Znbi60 ap 1a ont “ae 3 Figure 5.6 Werd-Leonard speed control system. Replacing &,, by E.. we obtain = Zn0160, Thats HETR A 60: 1 = ON) (approx) BRR AIGECA where 11 = speed of rotation [r/min] E, = armature voltage [V] = total number of armature conductors This important equation shows that the speed of the motor is directly proportional to the armature supply voltage and inversely proportional co the flux per pole. We will now study how this equation is applied, 5.5 Armature speed control Accoading (@ Eq, 5.7. if the flux per pole dis kept constant (permanent magnet field or field with fixed excitation), the speed depends only upon the armature vollage E., By raising oF lowering E., the ‘motor speed will rise and fallin proportion, In practice, we can vary E, by connecting the ovor armature M to a separately excited variable voltage de generator G (Fig. 5.6). The field excita tion oF the motor is kept constant, but the generator ‘excitation J, can be varied from zero to maximum and even reversed. The generator output voltage E, can therefere be varied trom zero to maxi- mum, with either positive or negative polarity ‘Consequently. he motor speed can be varied from ze10 1 maximum in either direction, Note that the senerator is driven by an ae motor connected 10 3 FAGILIAD DE XS.N 2A mills, high-rise elevators, mine DIRECT-CURRENT MOTORS 101 Ines) 3-phase line. This method ef speed control, known, as the Ward-Leonard system, is found in steel and paper mills In modern installations the generator is often re= placed by a high-power electronic converter th changes the ac power of the electrical utility to de by electronic means. The Ward-Leonard system is more than just a simple way of applying a vetiable de voltage to the armature oF a de motor, lean actually force the mo- tor to develop the torque and speed required by the load. For example, suppose F, is adjusted to be slightly higher than the czmf £,, of the motor. ‘Current will then flow in the direction shown in Fig. 5.6, and the motor develops & positive torque. The armature of the motor absorbs power because / ows into the positive terminal Now, suppose we reduce Eby reducing the erator excitation, As soon as E,hecomes fess tha E,, current [reverses, Asa result (1) the motor torque reverses and (2) the armature of the motor defivers powerto generator G, In eect the de motor suddenly becomes a renerator G suddenly be- ceomes a motor, The electrie power that the de motor snow delivers to G is derived at the expense of the ki netic energy of the rapidly decelerating armature and its connected mechanical load, Thus, by reducing E. the motor is suddenly forced to low down, ‘What happens to the de power received by gen- ‘erator G? When G receives electric power, it oper- tes asa motor, drivi c’chronous generator! AS a result, its own ae motoras an asyn- ae power is Fed The asyochconas generator is xplsined in Chae 1 102 ELECTRICAL MACHINES AND TRANSFORMERS back into the Tine that normally Feeds the ae motor. The fact that power can be seoered this way tauakes the Warul-Leonard system very elficient, and constitutes another ofits advantages, Example 5-3 —_ A 2000 kW. 500 Y, variable-speed mot by a 2504) KW generator. using a Ward-Leonard ceoniral system shown in Fig. 56, ‘Phe total resis nce of the motor and 10 mQ2, The motor turns at nominal speed of 300) min, when E, is 500 V. Catena a, The motor torque and speed when = 400 Vand £, = 380V bb. The motor torque and speed when = 350 Vand E, = 380 Solusion a4. The armature current is T= (BBR = (400 ~ 380V0.01 = 200A The power fo the motor armature is P= Bd = 380 2000 = 7608W ‘The motor speed is n= (380 V/S00 V) X 300 = 228 r/min The motor torque is T~9.S5Pin = (9.55 X 760 000/228 = SKN b. Because £, 228 renin, The armature current is Fe ER = 30008 380 V. the motor speed is still = 380/001 he current isn tive and so it flows in re- sonsequently. the motor torqu Power returned by the motor 1 the and the 10. mQ resistance: nerator P= Eg = 380 X 3000 = 1140 KW jorque developed by the motor 955PIn = (0.85 * 1 140.000)228 =478kNn ‘The speed of the motor and its connected me- chanical load will rapidly drop under the intluer Of this electromechanical braking torque, Rheostat Speed Control Another the speed of a de motor ist phice a rheost ries with the armaure (Fig. 5.7). The current in the rheostat produces a voltage drop which subtracts from the fixed source voltage E, yielding a smaller supply voltage actoss the armature. This method en- ables us to reduce the speed below its nominal speed. IL s only recommencled for small motors be- ‘cause ol of power and heat is wasted in the theo Stat. and the overall efficiency is low. Furthermore the speed regulation is poor. even for a fixed setting ‘ofthe rheosta, In effet, the JR drop across the rheo- sat increases as the armature eurrent inereases. This. produces a substantial drop in speed with increasing ‘mechanical load, ay 10 control ls | freer L shunt Wels Figure 5.7 ‘Armature speed control using @ sheostat 5.6 Field speed control According to Eg. 5.7 We can also vary the speed of ade motor by varying the field us ®. Let us now keep the urmature voltage E, constant so that the ‘numerator in Eq, 5.7 is constant. Consequently. the motor speed now changes in inverse proportion t the flux «P: if we inerease the flux the speed will drop, and view versa. This method of speed control is frequently used ‘when the motor has to run above its rated speed, called base speed, To control the flux (and hence, the speed), we connect a rheostat Ry in series with the field (Fig. 5.8), Touunderstand this method of speed control. sup pose that the motor in Fig, 5.8a is initially running at constant speed. The counter-emf Eis slightly Jess than the armature supply voltage E.. due to the IR drop in the armature. If we suddenly increase the resistance of the rheostat, both the exciting current J, and the flux & will diminish, This immediately reduces the cemt (© jump to a much higher value, The current causing the armature current f oo rate isd 0 1 + se armature current Pow Figure 5.8 ‘Schematic diagram of a shunt motor including the field heostat ». Torque-speed and torque-current characteristic of a shunt motor DIRECT-CURRENT MOTORS 103 changes dramatically because its value depends upon the very sinall difference between E,and E,, Despite the weaker field, the motor develops 3 greater torque than before. It will accelerate until E, is again almost equal to & Clearly. to develop the same E,, with a weaker flux, the motor must turn faster. We can therefore raise the motor speed above its nominal valle by in- twoducing « resistance in series with the field. For shunt-wound motors. this method of speed control ‘enables high-speedibase-speed ratios as hi 1. Broader speed ranges tend to produce instability and poor commutation, Under drop to dangerously low values. For example, ifthe ‘exciting current of a shunt motor is interrupted ace’ dentally, the only fux remaining is that due to the re- ‘manent magnetism in the poles,® This flux isso small that the motor has to rotate att dangerously high speed to induce the required cemt. inoxluced to prevent such runaway conditions as310 tain abnormal conditions, the flux may 5.7 Shunt motor under load Consider a de motor running at no-load. Ifa me- chanical load is suddenly applied to the shalt, the small no-load current does not produce enough tonque to carry the load and the motor begins to slow down, This causes the cemf to diminish, re- sulting in a higher current and a corresponding higher torque. When the torque developed by the motor is exactly equal to the torque imposed by the ‘mechanical load, then, and only then, will the speed remain constant (see Section 3.11). To sum up. ay the mechanical load increases, the rises and the speed crops. ‘The speed of a shunt motor stays relatively con. stant from no-Toad (0 full-foud. fa small motors it only drops by 10-t0 15 perent when full-load is Teter eid! again ya used: However the IEEE Stand Ditionar of Electric and Eletonics Terms sates.” na sete ici, he remanent indo wil eae ei induction: if hee ate i gaps wil ees than the residual indo. re noe zaps i he vane induction Ws ELECTRICAL MACHINES AND TRANSFORMERS applied. In big machines. the drop is even less. due in part. tothe very low armature resistance. By ad- justing the field rheostar, the speed can. of course. bbe ep absolutely constant a the load changes. Typical torque-speed and tonque-current chi teristics of a shunt motor are Shown in Fig, 5.86. The speed. torque and current are giver fa peesnit values. The toryu is diteetly proportional to the ar mature current. Furthermore, the speed changes only from 1.1 pu to 0.9 pu as the tore increases trom 0 puto 2 pu. Example 5-4 ———__ A shunt motor rotating at 1500 r/min is fed by a 120 V source #Fig, $-ba), The fine current is SLA find the shunt-field resistance is 120.0. 1F the are esistance is 0.1 0, caleutate the following: mature a. The current bb. The count ‘c. The mechanical power developed by the motor Solution: a The field eun 4 rt (Fig. 5.98) is = IO WIDO=1A ‘The armature current is =s1 b. The voltage weross the armature is B= 120 ‘Voltage drop due to armature resistance is IR = 50x 0.1~5V ‘The cemf generated by the armature is E,= 10-5=115V «. The total power supplied 1» the motor is P,= ED= 120 x 51 6120W Power absorbed by the armature is, P= EI = 120 50-= 6000 W Power dissipated in the armature is 50° x0 Pai 250 W 1 02e ty 0.8 Figure 5.9 See Example 5.4 Mechanical power developed by the armature is P = 6000 ~ 250 = 5750 W equivalent to 5750746 = 7.7 hp) The actual mechanical output is slightly fess than 5750 W because some of the mechanical power is dissipated in bearing friction losses. in windage losses. and in armature iron losses, 5.8 Series motor A series motor ix identical in consteuction 10 a shunt motor except for the field. The Held is con: nected in series with the armature and must, there: fore, carry the full armature current (Fig, 5, 10a). This series field is composed of a few turns of wire having a cross section sufficiently large to earry the current Although the construction is simi the proper Jes ofa series motor are completely different from those ofa shunt motor. In a shunt motor. the Muy F per pole is constant at all loads because the shunt Field is connected to the fine. But in a series motor the flux per pole depends upon the armature eurrent and, hence. upon the yal. When the current is large, the fluy is large and vice vers. Despite these differences. the same basie principles and equations apply to both machines. © to) Figure 5.10, 4, Series motor eannection diagram. Schematic diagram of a series motor. When a seres motor operutes at full-Toad, the ux per pole isthe same as that of a shunt motor oF identical power and speed. However, when the ‘aries motor stats up. the armature current is higher ‘han normal, with the result that the flux per pole is also greater than normal. follows that the Starting torgue ofa series motor is considerably that of'a shunt notor. This can be seen by compar- ing the T versus J curves of Figs, 5.8 and 5.11 On the other hand, if the motor operates at less ‘than foll-load, the armature current and the flux per pole than normal, The weaker field ‘cases the spel to tise in the same way as it would fora shunt motor with a weak Shunt field, For ex- ample. ifthe load current of a series motor drops to hulls normal value, the flux diminishes by half and sthe speed doubles, Obviously. if the load is small, the speed may rise to dangerously high values. For this ezason We never permit a series Motor to Oper ‘teat no-Foad. I: tends to run away, and the resulting centrifugal foress could tear the windings out of the armature and destroy the machine. 5.9 Series motor speed control When a series motor carries load, its speed may have to be adjusted slightly. Thus. the speed ean be yee in parallel increased by placing a low resis With the series field, The field current is then smaller than before, which produces a drop in flux and an increase in speed. DIRECT-CURRENT MOTORS 105 » speed > stmature current! Figure 5.11 Typical speed-torque and current-torque characteristic ‘of a series motor Conversely. the speed muy’ be lowered by con- necting an external resistor in series with the a ture and the field. The total JR drop across the re sistor and field reduces the armature supply ind So the Spee! must fall. Typical torque-speed and worque-current chara: teristics are shown in Fig, 511, They are quite dif ferent from the shunt motor sharacterities, Fig. 5.8b. voltage. Example 5 — A.15 hp. 240 V. 1780 r/min de series motor has a full-load rated current of 54 A. Its operating charac teristics are given by the per-anit curves of Fig. $.11 Caleutate a, The current and speed wien the load torque is 24m b. The efficiency under these conditions Solution a, We frst establish the base pow. and base current of the motor, They correspond to the full-load ratings as follows: hase speed, 106 ELECTRICAL MACHINES AND TRANSFORMERS Py = IShp = 15 746 = 11 190W ny = L780 simi. y= SA “The base tongue is. therefore, 9.55 Py 9.55. 11 19011 780 =60Nm A\ Toad torque of 24 N-m corresponds to a per- Unit torque of Tipu) = 2460 = 04 Referring to Fig. 5.11 asenque of 04 pu is at tained at a speed of 1.4 pu. Thus. the speed is = pay X sig = LAX 1780. = 2492 clmin From the T'vs / curve, a torque of 0.4 pu re- quires a current of 0.6 pu. Consequently. the load current is ’ Npu) X fy = 0.6 x S4= 324A b. To calculate the efficiency, we have to know P,, 4 = TIT W = 6263 W 1) = PJP, = 6263/7776 = 0.805 oF 80.5% 5.10 Applications of the series motor Series motors are used on equipment requiring a high torque. They are also used to drive devices ‘which must run at high speed at light loads. The series motor is particularly well adapted for ractiow Purposes. suck as i electric trains. Acceleration is rapid because the torque is high at low speeds Furthermore, the series motor automatical down as the train goes up a grade yet turns at top speed on flat ground. The power of a series motor tends o be constant, because high torgue is uecom= panied by low speed and vice versa, Series motors are also used in electrie cranes and hoists: light lols are lifted quickly and heavy loads more slowly, 5.11 Compound motor A compound de motor carries both a series field and shunt field, In a cumulative compound motor. the mmm of the two fields add, The shunt field iy always stronger than the series Fie. Fig. 5.12 shows the connection and schemati diagrams of a compound motor. When the niotor runs at no-load, the armature current Jn the seties ‘winding is low and the mmf of the series fret is negligible. However, the shunt field is fully excited by current /, and so the motor behaves like a shunt machine: it does not tend fo run away at no-load, As the Toad increases, the mmf ofthe series field inereases but the mmf of the shunt field remains constant, The total mmf (and the resulting Mux per pole) is therefore greater under load than at no-load ‘The motor speed falls with increasing load and the speed drop from no-load to full-load is generally between 10 percent and 30 percent stunt | wer | tes toy Figure 6.12 ‘2. Connection diagram of a de compound moter. ». Schematic diagram ofthe motor. Figure 5.13 ‘ypca speed versus torque characterises of various de motors. DIRECE-CURRENT MOTORS 107 It the series field is connected so that it opposes the shunt field, we obtain a differeatial compound rotor In such a motor the total mmf decreases with ineseasing lod, The speed rises as the load in creases, and this wy lead to instability. The differ ential compound motor hats very few applications. Fig, 5.13 shows the typical torques ‘of shunt, compound and series motors on ap basis. Fig. 5.14 shows a typical application of de motors in a steel mil, 5.12 Reversing the direction of rotation ‘To revere the direction of rotation of @ de motor, we must reverse either (1) the armature connections or (2) both the shunt and series field connections. The in- (erpoles are considered to form part of the armature hhange in connections is shown in Fig. 5.15 Figure 5.14 Hot stip finishing mill composed of 6 stands, each driven by @ 2500 KW de motor. The wide stee! strip is delivered to he runout table [et foreground) driven by 181 de motors, each rated 3 KW. (Courtesy of Ganeral Electric) 108 ELECTRICAL MACHINES AND TRANSFORMERS oo w Figure 8.15 2, Original connections of a compound motor el , Rloversing the armature connections to reverse the direction of rotation ‘6. Reversing the ld connections to reverse the direction of rotation 5.13 Starting a shunt motor 1 we apply full voltage 10 stationary shunt motor, the starting current inthe armature will be very high nd we run the risk of 4a, Burning out the armature: b. Daragiag the commutator and brushes. due 10 heavy sparking €. Overloading the feeder, 4. Snapping off the shat due to mechanical shock: te. Damaging the driven equipment because of the sudden mechanical hammerblow. All de motors must, therefore, be provided with ‘8 means to limit the starting current 10 reasonable values. usually between 1.5 and twice full-load cur rent, One solution is t connect a sheostat in series with the armature. The resistance is gradually res duced ay the motor accelerates and is eventually eliminated entirely, whee the machine has attained full speed. Toulay. electronic methods are often used 10, limit the starting current and to provide speed 5.14 Face-plate starter Fig. 5.16 shows the schematic diagram of a manual face-plate starter for a shunt motor. Bare copper contacts are connected to current-limitin Ry, Roe Rand Ry, Conducting arm 1 sweeps across the contacts when its pulled to the Fight by means cf insulated handle 2, In the position shown, the arm touches dead copper comact M and the motor circuit is open, As we draw the handle to the right, the conducting arm first touches fixed contact N ‘The su, y voltage E, immediately causer fal field currene /, co tTow, but the armacure current is limited by the four resistors in the starter box. The motor begins to turn and. as the cenat E,, builds up, the armature current gradually falls, When the motor speed ceases to rise any more. the arm is pulled tothe next contact thereby removing resistor Ry from the armature circu, The current invediety jumps toa niger value and the motor quickly accelerates tothe neXt higher speed. When the speed again levels of. we move to the next contact. and so forth, until the ari Finally touches the last contact. The arm is m ‘tically held inthis position by a smal electromag net 4, which isin series with the shunt field resistors DIRECT-CURRENT MOTORS 109 Figure 5.16 Manual face-plate starter for a shunt motor. Ifthe supply voltage is suddenly interrupted, or if the field excitation should accidentally be cut, the electromagnet releases the arm. allowing i to return to its dead position. under the pull of spring 3. This safety Teature prevents the motor from restarting un- expectedly when the supply voltage is eeestablished. 5.15 Stopping a motor One is inclined to believe that stopping a de motor is «simple, almost trivial, operation. Unfortunately, this is not always true, When a lange de motor is coupled loaheavy inertia loud, it may take an hour or more for the system £0 come 10 a halt, For many reasons such lengthy deceleration time is often unacceptable and, under ese cireumstances, we must apply a braking togque fo ensure a vapid stop. One way to brake the motor is by simple mechanical friction, in the same ‘way we slop car: A more elegant method consists of circulating a reveyse current in the armature, so as 10 brake the motor electrically. Two methods are em= ployed to ereute Such an electromechanical brake: (1) ‘dynamic braking and (2) plugging. 5.16 Dynamic braking Consider a shunt motor whose field is directly con- routed 19 a source and whose armature is con- nected to the same source by means of a double tow switeh, The switch connects the armature (© wither the line orton external resistor (Fig. 5.17), When the motor is running normally, the direc- tion ofthe armature current /, and the polarity of the ccomf £,,are as shown in Fig. 5,17a. Ni armature JR drop, &, is equal to Ey If we suddenly open the switeh (Fig. 5.17b), the ‘motor continues to turn, but its speed will gradually dcop due 10 Iriction and windage losses, On the bother hand, because the shuot field is still excited, induced voltage E,, continues to exist, falling 38 the same rate as the speed. In essence. the motor is now aa generator whose armature is on open-circuit. Let us close the switch om the second set of con: lets so that the armature is suddenly connected to the external resistor (Fig, 5.17e). Voltage E, will ime mediately produce an armature current fs, However: this current flows in the opposite direction to the original current). IC follows that reverse torque is developed whose magnitude depends upon fs. The reverse torque brings the machine 40 a rapid, but very smooth stop. Figure 5.17a ‘Armature connected to a de source E,. 110. ELECTRICAL MACHIVES AND TRANSFORMERS 7 iL} Figure 5.176 ‘Armature on open circuit generating a voltage E,. Figure 5.176 Dynami braking. In practice, resistor Ris chosen so tha the initial braking current is about twice the rated motor cur rent, The intial braking torgue is then twice the nor imal torque of the motor AAs the motor slows dow, the gradual decrease in E,, produces a comesponding decrease in fy Consequently, the braking torque becomes smaller and smaller, finally becoming zero when the arma. ture ceases to turn, The speed drops quickly at first and then more slowly, as the armature comes to a halt, The speed decrewses exponentially, somewhat like the voltage aeross a discharging capacitor ‘Consequently. the speed decreases by half in equal Intervals of time 7,. To illustrate the usefulness of dynamic braking, Fig. 5.18 compares the speed: time curves for 3 motor equipped with dynamic braking and ene ghae simply coasts to a stop. 5.17 Plugging We can slop the motor even more rapidly by using. ‘4 method called plugging. I consists of suddenly dynam braking Figure 5.18 ‘Speed versus time curves for various braking methods. reversing the armature current by teversing the te iminals of the source (Fig. 5.194), Under normal motor conditions, armature eur: rent J is given by A> EB ER, where R, isthe armature resistance. If we suddenly reverse the terminals of the souree. the net voltage acting on the armature circuit becomes (E,, + Ey) The so-called counteremf £,, of the armature is no Tonger counter to anything bor actwally adds to the supply voltage E,. This net voltage would produce aan enormous reverse current, perhaps SQ times greater than the full-load armature current, This current would initiate an are around the commute tor. destroying segments, brushes. and suppor, even before the line cireuir breakers coutd open, Figure 5.198 Armature connected fo de source E.. Figure 5.19b Plugging. ‘To prevent such a catasvophe, we must limit the reverse current by introducing a resistor R in series with the reversing circuit (Fig, 5.196). As in dynamic braking, the resistor is designed to limit the initial braking current /> 0 about twice full-load current With this plugging circuit, a reverse torgpe is devel: ‘oped even when the armature has come toa stop. In clfect at zeto speed, £,, = 0, but fy = EJR, which is shout one-half its initial value, As soon as the motor Mops. we must immediately open the armature cic cuit otherwise it will begin to un in reverse, Circuit imerruption is usually controlled by an automatic null-speed device mounted on the motor sha The curves of Fig. 5.18 enable us to compare plugging and dynamic braking for the same initial braking current, Note that plugging stops the motor completely alter an interval 27,,On the other hand, if dynamic braking is used, the speed is sill 25 per- cent of its original value at this time, Nevertheless, the comparative simplicity of dynamic braking ren- ders it more popular in most applications 5.18 Dynamic braking and mechanical time constant Wementioned that the speed decreases exponentially with time when a de motor is stopped by dynamic braking. We can therefore speak of a mechanical ‘ime constant T in much the same way we speak of the electrical time constant of a capacitor that dis- charges into resistor In essence, T is the time it takes for the speed of the motor to fall 10 36.8 percent of its initial DIRECT-CURRENT MOTORS | value, However. it is much easier t draw the speed-time curves by defining a new time con- stant T,, which is the time for the speed to de Crease 10 50 percent ofits original value. There is a direct mathematical relationship hetween the conventional time constant T and the half-time constant T,. Its given by T,, = 0.6937 5.8) We can prove that this mechanical time constant is given by Jn? IBLSP, Be) T= time For the motor speed to fall 10 ‘one-half its previous value [s} moment of inertia of the rotating parts, eferred to the motor shaft ikem} ny = initial speed of the motor when braking Starts [r/min P, = initial power delivered by the motor fo the braking resistor |W] 1315 =a constant fexuct value = (30/m)"Alog.21 (0,693 = a constant Jexact valu J Thisequation is based upon the assumption that the braking effect is entirely due to the energy dissi- pated in the braking resistor. In oor is subjected to an extra braking (orque duet se and iriction, and so the braking time will bre less than that given by Ey. 5.9. neral, the Example $0, ————________ A225 kW (~ 300 hp), 250 V, 1280 r/min de motor has windage. friction, and iron losses of 8 kW. It drives a large flywheel and the total moment of in- certia of the ywheel and armature is 177 kg-m?. The motor is connected to a 210 V de source, and its speed is 1280 r/min just before the armature is switched across a braking resistor of 02 0 Cateulare a, The mechanical time constant 7, of the bakin: system b. The time for the motor speed to drop to 20 r/min 112 BLECTRICAL MACHINES AND TRANSFORSIERS ce. The time for the speed to drop to 20 r/min if the only braking force is that due to the windage, friction, and iron losses Sohurion a4, We note thatthe armature voltage is 210 V and the speed is 1280 elmin, Wohen the armature is switched to the brak ing resistor, the induced voltage is sill very close to 210 V, The initial power delivered to the resistor is P, = F'ik = 210°0.2 = 220 500. ‘The time constant T, is T= Jag 13L8 P,) 177 1280" IB1S x 220500 = los (59) 'b, The motor speed drops by 50 percent every 105. ‘The speed versus time curve Follows the so- quence given below: speed (r/min) time(s) 1280 0 610 10 320 20 (60 30 80 40. 40 50 20 60 ‘The speed of the motor drops to 20 rimin alter an jagervat of 60. cc. The initial windage. fiction, and iron tosses are 8 KW. These losses do not vary with speed in exactly the same way as do the losses in a brak- ing resistor, However, the behavior is compara bic. which enables us to make a rough estimate of the braking time. We have ny = 1280p, = 8000 ‘The new time constant is T= In HIBS Py) 17T x 12807) 131.5 8000) 276s 4.6 min ‘The stopping time inereases in proportion to the time constant. Consequently. the time to reach 20 r/min is approximately 1 = QT6I10) x 60 = 28 min 1656 s This braking time is 28 times longer than when ddynamie braking is used. Theoretically, a motor which is dynamically braked never comes to a complete stop. In practice however, we can assume that the machine stops al- ter an interval equal to 5 7, seconds. IF the motor is plugged, the stopping time has a definite Value given by or, (5.10) where 1, = Stopping time using plugging Is} T, = time constant 2s given in Ei, 5 (sh Example 5-7 ‘The motor in Example 5-6 is plugged. and the brak ing resistor is inereased to 04 0, so that the initial braking current is the same as hefore, Cateulase a. The initial braking current and braking power b. The stopping time Solution ‘The net voltage acting across the resistor is Em E+E, 210 = 420 210. ‘The initial braking current is 1, = EIR = 42004 10504 The inital braking power is P, 210 x 1050 ~ 220.5 kW According to Eq, 5.9, T, has the sume value as betire T= 10s ‘The time to come to a complete stop is = 27, = 20s 5.19 Armature reaction Until now we have assumed thatthe only mmf act ingina de motor is that due tothe field, However, the current flowing in the armature conductors also ere aesarmagnetomotive force that distorts and weakens the flux coming trom the poles. This distortion and Field weakening takes place in motors as well as in rerators, We recall that the magnetic action of the anmature aunt Is called armanure reaetion, 5.20 Flux distortion due to armature reaction When a motor runs at no-load, the small current lowing in the armature does not appreciably affect the flux ¢, coming from the poles (Fig. 5.20). But ‘when the armature carve its normal current, it pro duces it Srong_magnetomotive force which, iP it acted alone, would create a fluy ¢s (Fig, 5.21), By superimposing &, and «P,, we obtain the resulting flux &, (Fig. 5.22). In our example the flux density increases under the left hall of the pole and it de «eases under the right hall, This unequal distribution Pxluces two important effects. First the neutral zone shifts toward the left (against the direction oF ratation), The result is poor commutation with sparking at the brushes. Second, due to the higher Mux density in pole tip A, saturation sets in, the increase oF flux under the left- hand side of the pote less than the decrease under the righthand side, Flux, at ful-toad is therefore Slightly Tess than flux, at no-load, For karge mat chines the decrease in flux may be ay much as 10 percent and it causes the speed to inerease with load Such a condition tends to be unstable: to eliminate the problem, we sometimes add a series field of one ‘or two turns 1o inerease the Hux under Toad, Such motors are said to have a abilizedshuunt winding. 5.21 Commutating poles To counter the effect of armature reaction and thereby improve commutation, we always place a secol conemataing poles between the main poles of ‘medium-and large-power de motors (Fig. 5.23). As DIRECE-CURRENT MOTORS 13 aul Tet Figure 5.20 Flux distribution in a motor tunning at no-load, Figure 5.21 Flux created by the ful-}oad armature current Figure 5.22 Resulting flux distribution in a motor running at fll: load. inthe case of a de generator, these narrow poles de: velop a magneteamative force equal and opposite (0 the mm ofthe armature so tia the respective may rnetomotive forces rise and fall together asthe load current varies. In practice, the mmf of the comme: tating poles is made slightly greater than that of the armature, Consequently. small flux subsists in the region of the commutating poles. The tux is de- signed to induce in the coil undergoing commu tion a voltage that is equal and opposite to the selt- induction voltage mentioned in Section 4.28. As a ly improved and takes tion 4.27, result, commutation is gre: place roughly as described in S Hd ELECTRICAL MACHINES AND TRANSFORME Figure 6.23 ‘The narrow commutating poles are placed between the main poles ofthis 6-pole motor. ‘The neutralization of the armature mmf is re stricted (0 the narrow zone covered by the commu tating poles, where commutation takes place. The ‘las distribution under the main poles unfortunately remains distorted. This creates no problem for mo- tors driving ordinary loads, But in special eases its necessary to add a compensating winding. a Feature ‘we will now describe. 5.22 Compensating winding Some de motors in the 100 KW to 10 MW (= 134 np to 13 400 hp) range employed in steel mills per- Forma series of rapid. heavy-duty opevations, They acceferate, decelerate. slop, and reverse, all in matter of seconds. The corresponding armature current increases, decreases, reverses in stepwise fashion, producing very sudden changes in arma- ture reaction, For such motors the commutating poles und series, Stabilizing windings do not adequately neutralize the armature mmf. Torgue and speed contr i ditticuk ‘under such transient conditions and flash-overs may ‘occur across the commutator. To eliminate this prob lem, special compensating windings are connected in series with the armature, They ure distributed in slots, ‘cut into the pole faces of the main field poles (Fig 5.24, Like commuting poles, these windings pro: duce a mmf equal and opposite to the mmf of the ar mature, However, because the windings are dist uted across the pole faces, the armature mmf is bucked from point to point, which eliminates the field distonion shown in Fig. 5.22. With compensa ing windings, the field distribution remains essen- tially undisturbed from no-load to full-load, retaining the general shape shown in Fig, 5.20. ‘The addition of compensating windings has a profound effect on the design and performance of & de eotor: |. A shorter air gap can be used because we no longer have to worry about the demagnetizing effect of the armature. A shorter gap means that the shunt field strength can be reduced and Dene the eoits ane smaller The inductance of the armature circuit i re duced by a factor of 4 or 5; consequently, the armature current can change more quickly and the motor gives a much beter response. This is particularly cae in big machines Armotor equipped with compensating windings ccan briefly develop 3 t0 4 times its rated torque, ‘The peak torque of an uncompensated motor is much lower whea the armature current is large. The reason is that de effective flux in the air gap Falls off rapidly with increasing current be. ‘cause of armature reaction. We conclude that compensating windings are essen: tial in large motors subjected to severe duty cycles. 5.23 Basics of variable speed control “The most important outputs of a de motor are its speed an tongue, Is use 0 determine the lis oF encl as the sped is increased rom zero o above Figure 5.24 DIRECT-CURRENT MOTORS IS Soepole dc motor having a compensating winding distributed in slots in the main poles, The machine also has 6 commatating poles. (Courtesy of General Electric Company) bse speed. In sodoing, the rated values of armature current, armature voltage, and field flex must not be exceeded, although lesser values may be used. In making our analysis, we assume an ideal sep rately excited shunt motor in which the armature re- 5.25), The armature volt wv the armature current Z the Tux the ‘exciting current J, and the speed 7 are all expressed in per-unit values. Thus, i the rated armature voltage E, happens tobe 240 V and the rated armature current J,i8600 A. they are both given a per-unit value of | Simitzly, the rated shunt field flux ¢, has a per-unit Sale of 1. The advantage of the per-unit approach is that it renders the rorgue-speed curve univers Thus, the per-unit torque 7 is given by the per unit fay &, times the per-unit armature eurent T2941, ou By the sume reasoning, vke per-unit armature voltage E, is equal tothe pee-unit speed times the permit flax dy Bane, is.) ‘The logis curve (Fig starting point of the torque-speed 1 isthe condition where the motor 116 BLECTRICAL MACHINES AND TRANSFORMERS Figure 6.25 Pecsunit circuit diagram 's , & + fy o ° 19 20 > speeds Figure 5.27 10 oe NN tos : o ° 10 125 20 ——» speed» Figure 5.28 develops rated torque (T= 1) atrated speed (1 = 1. The rated speed is often called base speed. In order to reduce the speed below base speed, we gradually reduce the armature voltage 10 zev while Aeeping the rated values of /,and constant at their per-nit value of 1, Applying Eq. (5.11). the ccortespondin torque T= 1X 1 = 1 Furthermore, according to Eq, (5.12), the per-unit voltage F, = % | =n. Figures 5.27 and 5.28 show the state oF E,, J, and 4, daring this phase of motor ‘operation, known as the constant torque mode. Next, to raise the speed above base speed, we re- alize that the armature voltage cannot be increased anymore because itis alteady at its rated level of 1 ‘The only solution is o keep E, at its rated level of | and reduce the flux, Referring to Eq, (5,12), this means that rP, = aad sa, = f/m. Thus, above base speed, the per-unit flux is equal to the recipro- cal of the per-unit speed. During this operatin ‘mode. the armature current can be kept at its rated level of 1. Recalling Eg. (5.11), it follows that T= thd, =(lin) X 1 =1/n, Consequently. above base speed, the perunit torque decreases as the recipro- cea ofthe per-unit speed. It is clear that since the per unit armature current and armature voltage are both equal to I during this phase, the power inpat to the motor is equal to 1. Having assumed an ideal mi chine. the per-unit mechanical power output is also equal to 1, which corresponds to sated power, That fs why the region above base speed is named the constant horsepower mode, We conclude that the ideal de shunt motor ean ‘operate anywhere within the limits of the torque speed curve depicted in Fig, 5.26. In practice, the actual torque-speed curve may di Jer considerably trom that shown in Fig. 5.26. The curve indicates an upper speed limit of 2 but some ‘machines can be pushed to limits of 3 and even 4. by reducing the flux accordingly. However, when the speed is raised above base speed, commutation prob: Jems develop and centrifugal forces may become dangerous. When te motor runs below base speed, the ventilation becomes poorer and the temperature tends 0 rise above its rated value, Consequently, the armature current must be reduced, which redaces the torque. Eventually, when the speed is zero, all forced ventilation ceases iad even the field current must be reduced 1 prevent overheating of the shunt field coils. As a result, the permissible stalled torque may only havea per-unit valve oF 0.25. The resulting prac tical torque-speed curve is shown in Fig, 5.29. ‘The drastie fall-off in torque as the speed ishes can be largely overcome by using an extemal blower tocoo! the motor. It delivers a constant stream ofa, no matter what the speed ofthe motor happens tobe, Under these conditions, the torque-speed curve approaches that shown in Fig. 5.26, E 3 3 5 d a2oscsonto 20 + so0ed Figure 5.29 Torque-speed curve of atypical de motor DIRECT-CURRENT MOTORS 117 5.24 Permanent magnet motors We have seen that shuntfield motors require coils and a field current to produce the Hlux. The energy consumed, the heat produced, and the relative large space taken up by the field poles are disad: vantages of a de motor. By using permanent ma nets instead of field coils, these disadvantages are overcome. The result is a smaller motor having a higher efficiency with the added henefit of never risking run-away due to field failure. A further advantage of using permanent magnets is thatthe effective air gap is increased many times. ‘The reason is that the magnets have & permeability that is nearly equal to that of aie, AS a result, the ar ‘mature mmf eannot ereate the intense Fie that is pos sible when softiton pole pieces are employed. Consequently, the field created by the magnets does. not become distorted, as shown in Fig. 9.22, Thus, the armature reaction is reduced and commutation is im proved, as well as the overload capacity of the motor ‘Afurther advantage i that the long ar gap reduces the inductance oF the armature and hence it vesponds ‘much more quickly to changes in armature curet Permanent magnet motors are particularly advan tageous in capacities below about § hp. The magnets Figure 5.30 Pormanent magnet motor rated 1.5 hp, 90 V, 2900 nmin, 14.5 A. Armature diameter: 73 mm; armature length 115 mm sos 20; commutator bars: 40; tums per coll 5; conductor size: No. 17 AWG, lap winding, Armature resistance at 20°C: o3en. (Courtesy of Baldor Electric Company) LIS ELECTRICAL MACHINES AND TRANSFORMERS fare ceramic or rue-earthicobalt alloys, Fig. 5.30 shows the construction oF 3 1.5 fp. 90 V, 2900 e/onin PAM motor. Its elongated armature ensures Tow iner- tiaand fast response when used in servo applications. ‘The only drawback of PM motors is the rela high cost of the magnets and the inability (© igher speeds by field weakening obiain Questions and Problems Practical Level 5-1 Name three types of de motors and make a sketch of the connections Explain what is meant by the generator ef= Feet in a motor 5-3 Whar derermines the magnitude and polar uy of the eounterem in ade motor? 5-4 The counteremf of a motor is always igily less than the applied armature vott- sige. Explain. 5:5. Name wo methods that are used to vary the speed of a de motor 5-6 Explain why the armature current of a shunt motor decreases asthe motor accelerates 5-7 Why isa starting resistor needed to bring a motor up to speed? 5-8 Show one way to reverse the direction of rotation of a Compound motor, 5-9 A230 V shunt motor has nominal arma- ture current of 60 A. If the armature resis lance is 0.15 2. calculate the following: a. The counter-emt [V] 'b. The power supplied to the samature [W] «6. The mechanical power developed by the mo- tor. [kW and Pp a, In Problem 5-9 caleulate the initial tating current i the motor is irectly connected tsross the 230 V Tine |, Calculate the value ofthe starting esistor needed to limit the inital eusrent 4 TIS A. Imermediate level S-IL_ The compound motor of Fig. $.12 has 1200 turns on the shunt winding and 25 turns on 5-13 Sd sls ‘the series winding, per pole. The shunt field bras a total resistance of 115 0, and the ‘nominal armature current is 23 A. Ifthe ‘motor is connected to a 230 V line, calcu late the Following: a. The mim per pole at ull-load . The mmf at no-load A separately excited de motor turns at 1200 iin when the armature is connected to 2 11S V souree, Calculate the armature volt- sage required so thatthe motor runs at 1500 iin, At 100 ¢imin, ‘The Sallowiag details are known about a 150 hp. 230 V, 435 rimin de shunt motor: nominal full-load current: 862 A insulation class: H. weight: 3400 ky. external diameter of the frame: 915 mm. length of frame: 1260 mm. 2, Calculate the total lowes and effivieney at full-toad b. Calculate the approximate shunt field exci ing curren ifthe shunt field causes 20 pr eit of the ttal losses «6 Caleulate the value oe armature resistance sa well asthe counteremf. Knowing that 50 percent ofthe tol losses at fl -load are due 4. IF we wish co attain a speed of 1100 min, What should he the approximate exciting ‘current? We wish to stop a 120 hp, 240 V. 400 r/min motor by using the dynamic brak- ing circuit shown in Fig. 5.17. IFthe nom- inal armature current is 400 A. calculate the following: 4. The value ofthe braking resistor Rif we ‘Want to limit she mexdmam braking eure 12S percent of iis nominal v bb. The braking power [KW] when the motor has decelerated 10 200i, 50 rin, O een. a, The motor in Problem 5-14 is now stopped bby using the plugging cireuit of Fig. 5.19, Calculate the new braking resistor R 90 that ‘the masishum braking curren is S00 A. b, Calculate the braking power [kW] when the ated 4s 200 min, 50 iin, motor has decel Ostmin «Compare the braking power developed at 200, ‘iin to the instantaneous power dissipated in resistor & Advanced level 5-16 The armature of a 225 kW, 1200 r/min mo- tor has a diameter of $59 mm and an axial length of 235 mm, Calculate the following: a. The approximate moment of inests, knowing that iron has a density of 7900 ken’ >, The kinetic energy of the armature alone wher it wens at 1200 efenin «The total kinetic enerey ofthe evolving parts ata speed of 600 smn, ithe J ofthe wind: lator is equal to the J eae ings and co fated in ca) 5-17 If we reduce the normal exciting current of practical shunt motor by $0 percent the speed increases, but it never doubles. Explain why. bearing in mind the saturation of the iron under normal excitation 5:18. The speed of a series motor drops with ris- ing temperature, while that of a shunt mo. tor increases. Explain. industrial Application 5-19. A permanent magnet motor equipped with -samariumn magnets loses 3% of its magnetism per 100°C increase in temper {ure The motor runs ata no-load speed of 2500 r/min when connected to a 150 V source in an ammibient temperature of 22°C. Estimate the speed ifthe motor is placed in a room where the ambient emperature is 40°C 5.20 DIRECE-CURRENT MOTORS 19 Refecring Fig. 5.30, calculate the follow a. The number of conductors on the armature by The value ofthe countercit at ul had The 0x per pole. miliwehers Wh A standard 20 hp. 240 V, 1500 ein sel: cooled de motor hasan elficieney of 88 A requirement has arisen whereby the mo tor should run at speeds ranging from 208 g/min to 1300 rain without verheat- ing. His devided to cool the machine by ine stalling an external blower and channeli the air by means of an ai duct. The highest expected ambient temperature is 30°C and the temperature ofthe air that exits the mo- tor should not exceed 35°C. Caleutate the capacity of the blower required, in cubic Feet per minute. (Hint: see Section 3.21.) A250 hp, 500 V de shunt motor draws a ‘nominal field current of 5.4 under rated load. The field resistance is 98.02, Calculate the ohmic value and power of the series re sistor needed so that the Field curreat drops to 4.5 A when the shunt field and resistor are connected to the 500 V source. AS hp de motor draws afield current of 0.68 A when the field is connected t0 a 150 V source. On the other hand, 3 500 hp ‘motor draws a field current of 4.3 A when the field is connected to a 300 V de sous. In each case. calculate the power required for the field asa per ntae ofthe rated power of the motor ‘What conclusions can you dravy from these results? CHAPTER 6 Efficiency and Heating of Electrical Machines 6.0 Introduction henever @ mutchine transforms energy from ‘one form to another, there is always @ certain loss. The loss takes place in the machine itself, ‘causing (1) an increase in temperature and (2) are duction in efficiency. From the standpoins of losses, electrical mae chines say fe divided into two groups: those that have revolving parts (motors, generators, ele.) and thove that _do not (transformers, reactors, etc.) Electrical and mechanical lossesare produced in o- tating machines, while only electrical losses. are produced in stationary machines In this chapter we analyze the losses in de ma also found in most eines, but the same losses a machines operating on alternating current, The ‘study oF power losses is important because it gives usa clue ay to how they may be reduced. We also cover the important topies of tempera: ture rise and the service life of electrical equipment ‘We show that both are related to the class of insult tion used and that these insulation classes have been standardized 6.1 Mechanical losses Mechanical losses are due 10 beuring. trietion brush friction, and windage. The friction losses depend upon the speed of uke smachine and upon the desige af the bearings, brushes, commutator, and. slip rings. Wind: speed and design of the cooling fan and on the tur bulence produced by the revolving parts, In the ab: sence of prior information, we usually conduct tests on the machine itself to determine the value of these mechanical losses. Rotating machines are usually cooled by an internal fan mounted on the motor shaft. Ht draws in cool air from the surroundings, blows it over the windings. and expels it again through suitable © Towses depend on the vents. In hostile environments, speciat cooling methods are sometimes used. as illustrated in Fig. 6.1 6.2 Electrical losses Elecrical losses are composed of the following EFFICIENCY AND HEATING OF ELECTRICAL MACHINES 124 1. Conductor FR losses isometimies called copper losses) 2. Brush losses. 2. Iron losses 1, Conductor Losses. The losses in conductor d pend upon its resistance and the square of the cur rent itcarfies. The resistance, in turn. depends upon the length, eross section, resistivity, and tempera ture of the conductor. The following equations en- able us to determine the resistance at any tempers (ure and for any material: Figure 6.1 Totaly enclosed, water-cooled, 450 kW, 3800 rimin mor tor for use in a hostile environment. Warm air inside the machine is blown upward and through a water-cooled heat exchanger, situated immediately above the Westinghouse nameplate Aer releasing its neat 10a set of water-cooled pipes, the cool ar reenters the ma: ccne by way of two rectangulat pipes leading nto the end bel. The cooling air therefore moves in a closed ‘rout, and he surrounding contaminated atmosphere never reaches the motor windings. The circular capped pipes located diagonally on the heat exchanger serve a cooing. water inlet and outlet respectively. (Courtesy of Westinghouse) R= t (6.1) on 4 p=oiltan (62 in which R= resistance of conductor |] = length of conductor [mi] 4 = eross section of conductor [n p= resistivity of eonductor at temperature ¢ am} Py = resistivity of conductor at 0°C [0m] f= temperature coetic OC HIPC] {= temperature of conductor [°C] int of resistance at The values of p and efor differen materials are listed in Appendix AX2. In de motors and genera- tors, copper losses occur in the armature, the series field, the shunt field, the commutating poles, and the compensating winding. These I°R losses show Up as heat, causin rise above ambient temperature Instead of using the FR equation, we sometimes prefer to express the losses in terms of the number fof watts per kilogram of conductor material. The en by the equation the conductor temperatures t0 losses are the P. 10007" pie 63) where , = specilic conductor power loss [Wikg] J = current density [A/mm"| ° {= density of the conductor |kg/m'] 1000 = constant, to take care of units resistivity of the conductor [n8m} According 10 this equation, the loss per unit mass is proportional t0 the square of the current density. For copper conductors, we use densities be tween 1.5 A/mny? and 6 A/mm®, The corresponding, losses vary from 5 W/kg to 90 Wikg (Fi. 6.2). The higher de to prevent an excessive temperature rise, ities require an efficient cooling system oper condtor Figure 6.2 Copper losses may be expressed in wats per kilo gram. 2, Brush Losses The PR losses in the brushes are negligible because the current density is only about 0.1 A/mm. which js far les than that used in cop= per. However, the conracr voltage drop between the brushes and commutator may produce significant losses. The drop varies from 0.8 V to 1.3 ¥, de nding on the type of brush, the applied pressure, and the brush current (Fig, 6.3). 3. Tron Losses Iron losses are produced in th mature of a de machine, They are due to hysteresis Nem! voltage op osyray Figure 6.3, ‘Brush contact voltage drop occurs between the brush face and commutator. and eddy currents, as previously explained in ions 2.27 and 2,30, [ron losses depend upon the magnetic flux density, the speed of rotation, the {quality of the steel, and the size of the armature, They typically range from 0.5 Woks to 20 Wikg. ‘The higher values occur in the armature teeth, where the Mux density may be as high as 1,77, The losses in the armature core are usually much lower. The losses can be minimized by annealing the steel (Fig, 64), Figure 6.4 ‘This 150 KW electric oven is used to anneal punched steal inations. This industrial process, carried out in a con ltofled atmosphere of 800°C, significantly reduces the iron losses. The laminations are seen as they leave the oven, (Courtesy of General Electric) EFFICIENCY AND HEATING OF ELECTRICAL MACHINES 128 Some inon losses are also produced in the pole faces, They are due o flux pulsations ereated as sue- cessive armature teeth and slots sweep across the poke face, Strange as it may seem. iron losses impose a me chanical drag on the armature, producin: wanical friction the same elfect as met Example 62) Ade machine turning at 875 smin earties an arma ture winding whose total weight is 40 kg. The cu rent density is A/mm” and the operating tempera- ture is 80°C, The total iron losses in the armature amount to 1100 W. Calculate a. The copper losses. b, The mechanical drag [N-m] due to the iron losses Solution a, Reterring to Table AX2 in the Appendix, the re Sistivity of copper at 80°C is p=p.(l + an 15.88 (1 + 0,004 27 * 80) = 21.3 nm The density of copper is $890 ke/m* The specitie power 10ss i (6.1) * x 21318890 1000 x 60 Wik Toval copper loss is P= 60 x 40 2400 W bh. The braking torque due to iron Towses, calculated from iTS 1100 = $75 719.55 T= 12. Nem or approximately 8.85 felbf 6.3 Losses as a function of load Ade motor running at no-load develops no useful power. However, it must absor’ some power from the Tine t continue to rotate, This no-lowl power overcomes the frietion, windage, and provides for the copper losses in the shunt field. The FR losses in the armature, seties field, and commutating field sre negligible because the no load current is seldcm more shan 5 percent of the rhominal full-load current. ‘As we Toad the muchine the current increases in the armature circuit, Consequently, the FR losses in ‘the armature circuit (consisting of the armature and all the other windings in series with it) wil rise. On the other hand, the no-load losses mentioned above remain essentially constant as the load inc unless the speed of the machine changes apprecia bly. It follows that the total Tosses inerease with load. Because they are converted into heat the tem perature of the machine rises progressively ss the Toad increases, However, the temperature must not exceed the ‘maximum allowable temperature of the insulation used in the machine. Consequently. there isa mit 10 the power that the machine ean deliver, This temper- ature-limited power enables us to establish the nom: inal or rared poser of the machine. A machine loaded beyond its nominal rating will usually ov heat. The insulation deteriorates more rapidly. which inevitably shortens the service life of the machine, armachine runs jarermintendts itean carry heavy overloads without overheating, provided that the op: erating time is short, Thus, a motor having a nominal rating of 10 KW can realy carry a foal of 12 KW for shor periods. However, for higher loads the capacity is limited by other factors, usually electrical, For in Stance, itis physically impossible for a generator rated at 10 KW to del ver an outpat of 100 kW, even for one millisecond. 6.4 Efficiency curve ‘The efficiency of a machine is the ratio of the us ful output power P, ‘0 the input power P, (Section 3.7). Furthermore, input power is equal to useful power plus the losses p, We can therefire write oP gy a=) P, f * 100 (6.4) p 124 ELECTRICAL MACHINES AND TRANSFORMERS where a = output power 1W1 = efficiency input power IW] p= loses IW] The following example shows how to calculate the A de compound motor having a rating of 10 KW. 1150 r/min, 230 V, 50.4. has the following losses at full Fou: bearing friction loss = 40Ww brush friction loss sow ‘windage loss = 200 (1) total mechanical foyses 200 W (2) inom losses. 420 (3) copper loss in the shunt field = 120W ‘copper losses at full Jaad: a, in the armature b, in the series field = inthe commutating wind total copper loss in the armature circuit at full }oad = wy 595 W Calculate the lowes and efficiency at no-load and at 25, 50, 75, 100, and 150 percent of the nom- inal rating of the machine. Draw a graph showing efficiency as a fumetion of mechanical oad (neglect the losses due te brush contact drop). Solution Nosload ‘The copper losses in the armature circuit are negligible at no-load. Consequently, the no- load losses are equal 10 the sum of the mechanical Tosses (1), the iron losses (2), and the shunt-field losses (3): no-load losses = 290 + 420 + 120 = 830 W. These losses remain essentially constant asthe Toud The elticien no-load because no use ful power is developed by the motor 25 percent load When the motor is Inaded (© 25 percent of its nominal rating, the armature cur rent is approximately 25 percent (or 1/4) ofits ful load value. Because the copper losses vary as the: square of the current, we have the following: ‘copper losses in the armature cireuit = (Ay x 595 = 37 no-load losses = 830 total losses, = 37 + 830 = 867 W Useful power developed by the motor at 25 perce load is P= 1OKW x (1/4) = 2500. W (~ 3,35 bp power supplied to the motor is P= 2500+ 867 = 3367 W and the efficiency is a= (Pgh) X 100 (62) = (2500/3367) 100 = 74% we find the losses at $0, 73 ad In the same way 100, and 150 percent of the nominal lo AL 50 percent Inad the losses ane (DY x 595 + 830 = 979 W AUTS percent load the losses are GI4Y? X 595 + 830 = 1165 W AL 100 percent load the losses are 595 + 830 = 1425 W ALSO percent load the losses are (1.5)° x 595 + 830 = 2169 W The elficieney calewiations for the various loads are listed in Table 64 and the results are shown _neuphically in Fig. 65. EFFICIENCY AND HEATING OF ELECTRICAL MACHINES . 100) - | | efficiency f 80 + + > Va | yo) 3 tution ew a7 — a A} 3 | C | jole |,t ° a 1 i ° oe ea 0 Figure 6.5 Losses ard eficioncy as a function of mechanical power. See Example 6.2 TABLEGA LOSSES AND EFFICIENCY OF A DC MOTOR: Tot Ouipat Tap J tosex_— power 2, power, —icieney ww iw tel ar) 830 0 2 867 3367 4 so 9 5980836 75 116s 668 $6.5 foo 1425 Go aas 8S 15000 17170 102169) srs ‘The efficiency curve rises sharply as the load in creases, Mattens oF over a broad range of power. and then slowty begins to fall. This is typical of the sifieieney curves of all electric motors, both ae and rie motor designers usually try to attain the iency at full-load In the above calculation of efficiency we could have included the losses due to brusb soltage drop. Assuming. constant drop. say. of 0.8 V per brush, the Ish lows at full-Ioad amounts 1008 VX 50 Ax 2 brushes = 80 WW AL SO percent load. the brush loss ‘would be 40 W. These losses, when added to the other ‘oxses, modify the efficiency curve only slightly Tis important to remember that at light loads the cfficieney of any motor is poor, Consequently, when selecting a motor to do a particular job, we should always choose one having a poser rating roughly equal to the load it has te drive. We can prove that the efficiency of any de ma- chine reaches a masimum at that load where the ar mature cizeatt capper losses are equal tothe no-load losses. In our example this corresponds 10 a total Joss of (830 + 830) = 1660 W. an output of 11 811 W (15.8 hp) and anelficiency of 87,68 percent. The reader may wish to cheek these results 6.5 Temperature rise The temperature rise of a machine or device is the Uifference beween the temperature of its warmest accessible part and the ambient temperature, It may be measured by simply using two thermometers. However, due to the practical difficulty of placing thermometer close tothe really warmest spot inside the machine. this method is seldom used. We usu. ally rely upon more sophis rouls. de- scribed in the following sections. ‘Temperature ise has a direct beating on the Nalohasa di cated power rating of machine or dev 126 ELECTRICAL MACHINES AND TRANSFORMERS rect bearing on its useful service life, Consequently temperature rise isa very important quantity 6.6 Life expectancy of electric equipment Apart from accidental electrical and mechanical Failures, the life expectancy of electrical apparatus is limited by the temperature of its insulation: The higher the temperature, the shorter its life. Tests ‘made on many insulating materials have shown that of electrical apparatus diminishes approximately by half every time the temperature increases by 10°C. This means that if motor has a normal life expectancy of eight years ata tempera ture of 105°C, it will have a service life of only four ‘years at a temperature of 115°C, of two yeurs at 125°C. and of only one year at 135°C The factors that contribute most to the deteriora- tion of insulators are (1) heat, (2) humidity, (3) vi- bration, (4) acidity, (5) oxidation, and (6) time (Fig, 6.6). Because of these fuctors, the state of the insu Tation changes gradually: it slowly begins to erys: tallize and the transformation takes place more rapidly as the temperature rises. the service li In crystallizing, organic insulators become bard ‘nd bitte. Eventually the slightest shock or mechan. ical vibration will cause them to break, Under nonmal conditions of operation, most organie insulators have a lle expectancy of eight o ten years provided that their temperature does not exceed 100°C. On the other hhand, some synth ers can withstand temper atures as high as 200°C for the same length of time. Low femperatures are just ay harmful as bi temperatures are, because the insulation tends to freeze and crack. Special synthetic organic insul tory have been developed, however, whieh retain their Mexibility at temperatures as low as ~60°C, 6.7 Thermal classification of insulators Committees and organizations that set standards* have grouped insulators into five classes, depending upon their ability to withstand heat. These classes correspond to the maximum temperature levels of 108°C. 130°C. 158°C. 180°C. and 220°C (formeriy * Such as IEEE. Undersiters Laboratories. Canalsn Standirds Association, r fo © Qe Lj © | Be if lan QO -Sy/ Figure 6.6 Factors that may shorten the service sie ofan insulator. EFFICIENCY AND HEATING OF ELECTRICAL MACHINES. 127 represented by the letters A,B, F, Hand R). This 40°C. This standardized temperature way estab- thermal classification (Table 6B) isa cornerstone in lished for the following reasons: the design and manufacture of electrical apparats, 1. Itenables electrical manufueturers to fore 68 Maximum ambient temperature the worst ambient temperature conditions that and hot-spot temperature rise their machines are likely to encounter. Standards. organizations have also established a 2, Ienables them fo standardize the size of their i ‘ambient temperature, which is usually ‘machines and to give performance guarantees. TABLE6B CLASSES OF INSULATION SYSTEMS Clos ieratve ample and dino 1os°¢ Mate Ik, and paper when suitably impregnated oF A coated oF when immersed ina dielgetic liquid such as oil, Other materials oF combinations of materials may be included in this class if by experience or accepted tests they can be shossn to have comparable thermal life at 108°C. Is or combinations of materials such 3s cotton, 30°C Materials or combinations of materials such as mica, glass fiber asbestos, et. with suitable honing B substances. Other materials or eombinstions oF materials may be included in this cass if by exper ‘or accepted tests they can be shows to have comparable thermal lite 130°C, 155°C Materials or combinations of matesials such as mica, glass fiber asbestos. et. with suitable honding F substances. Other materials or combinations oF materials may be included inthis cass if by experience nC mica, glass ‘or accepted tests they ean be shows to have comparable fife at 180°C Material or combinations of materiaks such as silicone ek H_ with suitable bonding substances such as appropriate silicone resins, Other materials or combinations of eF ashesos. ete. ‘materials may be included in this clas if by experience or aecepted tests they ean be shown to have comparable life at 180°C. 200°C Materials or combinations of materials which by experience or accepted tests can he shown to have N-the required thermal life at 200°C. 220°C Materials or combinations of materials which by experience or aecepted tests cum be shown to have the reyuited thermal ite at 220°C 240°C Materials or combinations of materials whieh by experience or acespted tests can be shown to have She required thermal ie at 240°C. above Materials consisting entirely of mica, porcelain, glass quar. and similar inorganic materials. Other 240°C materials or combinations of materials may be included in this class if by experience or accepted tests C__they can be shown to have the required thermal if at temperatures above 240°C. ‘The abwne insulation classes indicate a normal fife expectaney of 20.00 h to 40 000 hat the stated temperature. This inplies tht electsical equipment insulated with class A insulation system would probably lst For 210 5 years if oper- ated continuously at 105°C. Note tho this clasiticaton assumes ha the insulation system is no in contact with wort sit, humid, or dusty atmospheres Fora complete explanation of insulation classes, insulation systems, the companion EEE Standards Publications Nos, 96,97, 98, 99, and 101, See UUndseiers Laboratories publication on insulation ssteny UL 1446, 1978, nd tomperature indies, see IEEE Std 1-1969 and o IEEE Std 117-1974 and bs ‘The temperature ofa machine varies from point 10 point, but there are places where the temperature is ‘warmer than anywhere else. This hottest-spot tem perature must not exceed the maximum allowable temperature ofthe particular class of insulation used Figure 6.7 shows the hot-spot temperature limits tor class A. B, F.and H insulation (curve 1). They are the temperature limits previously mentioned in Section 6.7. The maximum ambient temperature of 40°C is also shown (curve 3). The temperature dif fotence hetween curve | and curve 3 gives the mu ‘num permissible temperature rise for each insula tion class, This limiting temperate rise enables the manufacturer to establish the phys al size of the ELECTRICAL MACHINES AND TRANSFORMERS motor, relay, and so forth, he intends to put on the market. Thus, for class B insulation, the maximum allowable temperature rise is (130 ~ 40) = 90°C. To show hiow the temperature rise affects the size ‘of a machine, suppose a manufacturer has designed and builta LOW motor using elass B insulation. To fest the motor he places it in a constant ambient temperature of 40°C and loads it up until i delivers 10 KW of mechanical power. Special temperature detectors, located at strategie points inside the mar chine, record the temperature of the windings. After the temperatures fave stabilized (which may take several hours) the hottest temperature is noted, and this is called the kor-spor temperanure. IC the hot spot temperature so recorded is, say 147°C. the cm ware —~7> cas | wesc / 165°C | cus A Tee ware 7 / / oy i Class A i—S hot-spot A/a tense ise wre // au ® “ | a = oom, aetage ® 100°C temperature ise rena etd we wwe @ Figure 6.7 Timing ambient rompeatore “Typical imits of some de and ac industrial machines, according to the insulation class: (1) Shows the maximum permissible temperature of the insulation to obiain a reasonable service ite (2) Shows the maximum permissible temperature usin (9) Shows the limiting ambient temperature 1g the resistance method EFFICIENCY AND HEATING OF ELECTRICAL MACHINES 129 manufacturer would not be permitted t0 sell his product. The reason is that the temper (147 = 40°) = 107°C exceeds the maximum per- mnissible rise of 90°C for class B insulation, On the other hand, if the hottest-spot temps ture is only 100°C. the temperature rise is (100° — 10°) = 60°C. The manufacturer immediately per vives that he cam make more economical design amd still remain within the permissible temperature rise limits. For instance, he can recuce the conduc tor size until the hot-spot temperature rise is very close 10 90°C, Obviously, this reduces the Weight and cox of the windings. But the manufacturer also realizes that the reduced conductor size now en= ales him to reduce the size of the slots tum, reduces the amount of iron, By thus redesign- the motor. the manufacturer ultimately els up with s machine that operates within the permissible temperature rise limits and has the smallest possible physical size, as well as lowest cost In practice, itis not convenient to carry out per- fiymance tests in a controlled ambient temperature of 40°C. The motor is usually loaded (0 its rated ex pacity in much Tower (and more comfortable) am bien! temperatures. Toward this end. it has been es- tablished by. standards-setting bodies that. for testing purposes. the ambient temperature may lie anywhere between 10°C and 40°C. The hottest-spot tempersure is recorded a before, I the tempers lure rise under these conditions is equal to oF less than 90°C (for cs B insulation), the manufacturer isallowed to sell his produ. Example 6 A75 AW motor, insulated class F, operates at Tulle Toad in an ambient temperature of 32°C. ¥ the hot= spot temperature is 125°C. does the motor meet the ‘temperature standards? Soliton The hot-spot temperature rise is (28° = 32°) = 93°C According to Fig. 6.7, the permissible hot-spot ter= perature rise for cltss F insulation is (158° ~ 40°) 15°C. The motor eavily meets the temperature Standards. The manufacturer could red ‘ofthe motor and thereby markets more competitive proxlut 6.9 Temperature rise by the resistance method The hot-spot temperature vise is rather difficult 10 ‘measure because it has fo be taken atthe very inside fof a winding. This can be done by embedding a smal temperature detector, such as thermozouple fr thermistor, However, his direct methox! of mex hot-spot temperature is costly, and is only ied for larger machines, ‘To simplify matters, accepted standards permit second method of determining temperature rise, Mis based upon the average winding temperature. sured by resistance, rather than the hot-spot tempe ature. The maximum allowable 2 femperatures for the various insulation classes ae shown in curve 2, Fig. 6.7, For example in the ease of clas B insulation, an average winding tempera ture of 120°C is assumed to correspond to: hot-spot temperature of 130°C, Consequently. an aver temperature rise of 120° — 40°) = SU"C is assumed. to correspond to a hot-spot temperature rise of (130) 40°) = 90°C, ‘The averaye temperature of a winding is found by the resistance method. 1 consists oF measuring the winding resistance at a known windit perature, and measuring it ag chine is hot, For example. if the winding is made oF copper, we ean use the follow rived from Eqs. 6.1 and 6.2) to determine its 20 ‘erage temperature jeraye windin equation (de R, 1 = FE O34 + 1) = 234 63) R, fy = average temperature ofthe windi when hot [°C] 234 = a constant equall to Ha = 10,004 27 Ry = hot resistance of the winding [02] Ry ~ cold resistance of the winding [0] 1, = temperature of the wind rel when cold 10. PLECTRICAL MACHINES AND TRANSFORMERS Knowing the hot winding temperature by the re sistance method, we ean immediately eafeutate the corresponding temperature rise by subtracting the ambient temperature. IF this temperature rise falls within the permissible limit (80°C for class B insu: lation), the product is acceptable from a standards point of view. Note that when performance tests are ‘carried out using the resistance method, the ambient temperature must again lie between 10°C and 40°C. If the winding happens t0 be made of alurtinum Wire, Eg. 6.3 can still be used, but the number 234 ‘must be changed to 228. Example 6-4 — ‘A de motor that has been fle for several days in an ambient temperature of 19°C. is found to have a shunt-field resistance of 22.0. The motor then op= erates at full-load and, when ten bilized. the field resistance is found to be 30.0, The corresponding ambient temperature is 24°C. I the motor is built with class B insulation, ealeulate the atures have sta following: a. The load b. The full-load temperature rise by the resistance method & Whether the motor meets the temperature stan ards ze temperature of the winding, a full- Solution a. The average temperature of the shunt field at full-load is fy = (RUR)) O34 + 4) = 234 = (30/22) (234 + 19) = 234 = HC, b. The average temperature rise at full Toad is He = 24 = 87°C, The maximum allowable temperature rive by resistance Jor eluss B insulation is (120° ~ 40°) 80°C. Consequently, the motor does not meet the standards. Either its rating will have to be reduced, or the cooling system improved, before it eam be put on the market Alternatively, # may be rewound using class F insulation, As a very last resort, its size may hhave to be increased, A final word of caution: temperature rise stan- dards depend not only on the class of insulation, but also on the lype of apparatus (motor, transformer, relay, ete), the type of construction (drip-proof, to tally enclosed, etc.), and the field of application of the apparatus (commercial, industrial, naval. etc). Convequently. the pertinent standards must always be consulted before conducting a heat-run test on a specilic machine or device (Fig. 6.10), 6.10 Relationship between the speed and size of a machine Although maximum allowable temperature rise es tablishes the nominal power rating of a machine, is basie physical size depends upon the power and of ro Consider the 100 kW, 250 V, 2000 r/min gener: ator shown in Fig. 6.8. Suppose we have to build another generator having the same power and vot age, but running a half the speed. To generate the same voltage a half the speed, ‘we cither have to double the qumber of conductors ‘on the armature or double the flux trom the poles Consequently, we must either inerease the size of the armature, or increase the size of the poles. In practice, we inerease both. We conclude that for a ‘given power output, a low-speed machine is always, bigger than a high-speed machine (Fig, 6.9). This is true for both ac and de machines. Basically, the size ofa machine depends uniquely ‘upon is torque. Thus, a 100 kW, 2000 r/min motor Figure 6.8 100 KW, 2000 r/min motor; mass: 300 kg. ee EFFICIENCY AND HEATING OF ELECURICAL MACHINES 131 Figure 6.9 100 KW, 1000 r/min motor; mass: 500 kg has about the same physical size as a 10 KW motor ‘unning at 200 r/min because they develop the sa vorque. LLow-speed motors ane therefore much more costly than high-speed motors of equal power. Consequently For low-speed drives, itis often cheaper to use a small high-speed motor with a gear box that 10 use a large low-speed motor directly coupled to its load Questions and Problems Practical level 6-1 Name the losses in a de motor, 62 What causes iron losses and how can they be reduced? 63. Explain why the temperature of a machine increases as the load increases 6-4 What determines the power rating of a mae chine? 65. Iwe cover up the vents in a motor its out put power must be reduced. Explain. 66 Ifa motor operates ina cold environment, may we load it above its rated power? Why? 67 Name some of the factors that contribute to the deterioration of organic insulators, 68 A motor is buill with class H insulation, ‘What maximum hot-spot temperature can it withstand? Intermediate level 6-9 Ade motor connected to 240 V line ro du wanical output of 160 bp. Knowing that the losses are 12 kW. caleu- late the input power and the fine current 6-10 ALIS V de generator delivers 120 Atoa load. IF the generator has an efficieney of 81 percent, calculate the mechanical power needed to drive it [ap 6-14 Calculate the Full-load current of a 250 hp, 230 V de motor having an efficiency of 92 percent 6-12 A machine having class B insulation attains temperature of 208°C (by resistance) in a torrid ambient temperature of 180°C fa, What is the temperature rise? Di, Isthe machine running 10 hor and, if9. by how much? 6-13 The efficiency of a motor is always low when it operates at 10 percent of its nomi nal power rating. Explain 6-14 Calculate the efficiency of the motor in Example 6-2 when it delivers an output of Vp. 6-18 Aneleetrie motor driving a skip hoist with: draws 1.5 metric tons of minerals from a wench 20'm deep every 30 seconds, I the hoist has an overall efficiency of 94 per- cont, calculate the power output of the mo. tor in horsepower and in kilowats, 6-16 ‘Thermocouples are used to mesure the inter nal hot-spot winding temperature of a 1200 KW ae motor, insulated class FI the motor runs at fall-load, what is the maxiniu tem perature these detectors Should indicate in an ambient temperature of 40°C? 30°C? LC? 6-17 A.60)hp ae motor with class F insulation has a cold winding resistance of 12 at 23°C. When it runs at rated Joa in an am. bient temperature of 31°C. the hot windin; resistance is found to be 17.4.0. ‘a Calgulate the hot winding temperature, De. Caleulatethe temperature vse oF the motor 132 ELECTRICAL MACHINES AND TRANSFORMERS «6 Could the maracturerinerease the ame plate rating of the motor? Explain 6-18 Anelecttic motor has a normal life of ccight years when the ambient temperature is 30°C. IF it is installed in a location where the ambient temperature is 60°C, What is the new probable service life of the motor? 6-19 ANo. 10 round copper wire 210 m lon carries a current of 12.A. Knowing thal the temperature of the conductor is 105°C. cal culate the following: fa, The current density [A/a] '. The specific copper losses [Wikg] Advanced level 620 An afuminum conductor operates at a cur rent density of 2 A/mm a. Irthe conductor temperature is 120°C calcu late the specific losses [Wk] Express the eurtent density in circular mils per ampere. ‘The temperature rise of a motor is roughly proportional tos fosses. On the other hand, its efficiency is reasonably constant in the range between 50 percent and 150 percent of its nominal rating (see. for example, Fig. 6.5), Based on these facts, if a 20 KW motor has a full- load temperature rise oF 80°C, what power can it deliver at a temperature rise of 105°C? Anelectromagnet (insulated class A) situ- ated in a particularly hot location has a ser- vice life of two years. What is its expected life span it itis rewound using class F ins lation? An IT AW ae motor having class B insula tion would normally have a service life of 20.000 h, provided the winding temper ture by resistance does not exceed 120°C. By how many hours isthe service life re- duced if the motor runs for 3 hata temper- ature (by resistance) of 200°C? 621 622 6.23 Industrial application 6-24 6-26 6.29 A reel of No. 2/0 single copper conductor thas a resistance of 0.135 ohms ala temper: ature of 25°C. Calculate the approximate \Weight of the conductor in pounds The Table in Appendix AX3 tists che prop erties of commercially available copper conductors, In an electrical installation, its proposed to use a No, 4 AWG conductor in an area where the operating temperature of the conductor may be as high as 70°C. Using Eq. 6.2, calculate the resistance un. der these conditions of 2-conductor cable No. 4 AWG shat #27 meters long The shunt field ofa 4-pole de motor has 3 {otal resistance of 56 ohms at 25°C, By scraping off the insulation. itis found that the bare copper wire has a diameter of 0.04 inches. Determine the AWG wie size, and calculate the weight of the wire per pole, in kilograms. ‘The National Electrical Code allows a max- imum current of 65.A in a No. 6 gauge cop- per conductor. ype RW 75, A 420 ft eable is being used on a 240 V de circuit to carry current of 48 A, Assuming a maximum ‘operating temperature of 70°C. calculate the followi a, The poster fas, in watts, inthe 2-conductor ile >. The approximate voltage atthe load end it the vot service panel is 243 V. In Problem 6-27, if the voltage drop in the cable must not exceed 10 V when it is ear rying a current of 60 A, what minimum conductor size would you recommend? Assure a maximum operating temperature of 70°C, ‘Ade busbar 4 inches wide, 1/4 inch tl and 30 feet long carries a current of 2500 A, Calculate the voltage drop if the emperature of the busbar is 105°C. What is the power loss per meter? ck, EFFICIENCY AND HEATING OF ELECTRICAL MACHINES 138 6-30 Equation 6.3 gives the resistance/tempera- {ure relationship of copper conductors, namely, 1 = RUR, OM +14) = 234 Using the information given in Appendix. AX2, deduce similar equation for alu- ‘minum conductors. 6.31 The commutator of a 1.5 hp, 2-pole, 3000 min de motor has a diameter of 63 mm. Calculate the peripheral speed in feet per ‘minute and in miles per hour, 6.32 The Following information is given on the brushes used in the motor of Problem 6-31: number of brushes: 2 ccurrent per brush: 15.8 ‘brush dimensions: 5/8 in wide, 5/16 in thick, 3/4 in long. (The S/16 in». 5/8 in area isin contact with the commutator) x of brush: 0.0016 Qin resist brush pressure: 1.5 [bf brush comtaet drop: 1.2 V coefficient of friction: 0.2 Calculate the following: a. The resistance of the brush body in ohms b. The voltage drop in the brush body ‘c. The total voltage drop in one brush, including the contact voltage drop . The total electrical power loss (in watts) due 10 the two brushes ce. The frictional force of one brush rubbing against the commutator surface (in Ibf an newtons) The frictional energy expended by the two brushes when the commutator makes one revo- lution Gin joules) 18. The power loss due to Friction, given the speed ‘of 3000 r/min 1h. The total brush loss as percent of the 1.5 hp motor rating Figure 6.10 ‘This VARMECA® induction moto, including is variable-speed controller and gear reducer, is housed under a trans patent plastic cover. The entire unit is being water-sprayed to demonstrate is abilly 1o operate continuously under harsh environmental conditions. (Courtesy of Leroy Somer, a dvision of Emerson Electric)

You might also like